Keywords

Mathematics Subject Classification

1 Introduction

Suppose that \(p > 1, \frac{1} {p} + \frac{1} {q} = 1,f(x),g(y) \geq 0,f \in L^{p}(\mathbf{R}_{ +}),g \in L^{q}(\mathbf{R}_{ +})\),

$$\displaystyle{\vert \vert f\vert \vert _{p} = \left \{\int _{0}^{\infty }f^{p}(x)dx\right \}^{\frac{1} {p} } > 0,}$$

 | | g | |  q  > 0. We have the following Hardy–Hilbert’s integral inequality (cf. [1]):

$$\displaystyle{ \int _{0}^{\infty }\int _{ 0}^{\infty }\frac{f(x)g(y)} {x + y} dxdy < \frac{\pi } {\sin (\pi /p)}\vert \vert f\vert \vert _{p}\vert \vert g\vert \vert _{q}, }$$
(1)

where the constant factor \(\frac{\pi }{\sin (\pi /p)}\) is the best possible. If \(a_{m},b_{n} \geq 0,a =\{ a_{m}\}_{m=1}^{\infty }\in l^{p},b =\{ b_{n}\}_{n=1}^{\infty }\in l^{q}\),

$$\displaystyle{\vert \vert a\vert \vert _{p} = \left \{\sum _{m=1}^{\infty }a_{ m}^{p}\right \}^{\frac{1} {p} } > 0,}$$

 | | b | |  q  > 0, then we have the following discrete Hardy–Hilbert’s inequality with the same best constant \(\frac{\pi }{\sin (\pi /p)}:\)

$$\displaystyle{ \sum _{m=1}^{\infty }\sum _{ n=1}^{\infty } \frac{a_{m}b_{n}} {m + n} < \frac{\pi } {\sin (\pi /p)}\vert \vert a\vert \vert _{p}\vert \vert b\vert \vert _{q}. }$$
(2)

Inequalities (1) and (2) are important in Analysis and its applications (cf. [16]).

In 1998, by introducing an independent parameter λ ∈ (0, 1], Yang [7] gave an extension of (1) for p = q = 2. In 2009 and 2011, Yang [3, 4] gave some extensions of (1) and (2) as follows: If \(\lambda _{1},\lambda _{2} \in \mathbf{R = (-\infty,\infty )},\lambda _{1} +\lambda _{2} =\lambda,k_{\lambda }(x,y)\) is a nonnegative homogeneous function of degree −λ in \(\mathbf{R}_{+}^{2}\), with

$$\displaystyle\begin{array}{rcl} & k(\lambda _{1}) =\int _{ 0}^{\infty }k_{\lambda }(t,1)t^{\lambda _{1}-1}dt \in \mathbf{R}_{+} = (0,\infty ), & {}\\ & \phi (x) = x^{p(1-\lambda _{1})-1},\psi (y) = y^{q(1-\lambda _{2})-1}(x,y \in \mathbf{R}_{+}),& {}\\ \end{array}$$

f(x), g(y) ≥ 0, satisfying

$$\displaystyle{f \in L_{p,\phi }(\mathbf{R}_{+}) = \left \{f;\vert \vert f\vert \vert _{p,\phi }:= \left \{\int _{0}^{\infty }\phi (x)\vert f(x)\vert ^{p}dx\right \}^{\frac{1} {p} } < \infty \right \}\!\!,}$$

\(g \in L_{q,\psi }(\mathbf{R}_{+}),\vert \vert f\vert \vert _{p,\phi },\vert \vert g\vert \vert _{q,\psi } > 0\), then we have

$$\displaystyle{ \int _{0}^{\infty }\int _{ 0}^{\infty }k_{\lambda }(x,y)f(x)g(y)dxdy < k(\lambda _{ 1})\vert \vert f\vert \vert _{p,\phi }\vert \vert g\vert \vert _{q,\psi }, }$$
(3)

where the constant factor k(λ 1) is the best possible. Moreover, if k λ (x, y) is finite and \(k_{\lambda }(x,y)x^{\lambda _{1}-1}(k_{\lambda }(x,y)y^{\lambda _{2}-1})\) is strict decreasing with respect to x > 0(y > 0), then for a m,  b n  ≥ 0,

$$\displaystyle{a =\{ a_{m}\}_{m=1}^{\infty }\in l_{ p,\phi } = \left \{a;\vert \vert a\vert \vert _{p,\phi }:= \left \{\sum _{n=1}^{\infty }\phi (n)\vert a_{ n}\vert ^{p}\right \}^{\frac{1} {p} } < \infty \right \}\!\!,}$$

\(b =\{ b_{n}\}_{n=1}^{\infty }\in l_{q,\psi }\), \(\vert \vert a\vert \vert _{p,\phi },\vert \vert b\vert \vert _{q,\psi } > 0\), we have

$$\displaystyle{ \sum _{m=1}^{\infty }\sum _{ n=1}^{\infty }k_{\lambda }(m,n)a_{ m}b_{n} < k(\lambda _{1})\vert \vert a\vert \vert _{p,\phi }\vert \vert b\vert \vert _{q,\psi }, }$$
(4)

where the constant factor k(λ 1) is still the best possible.

Clearly, for \(\lambda = 1,k_{1}(x,y) = \frac{1} {x+y}\), \(\lambda _{1} = \frac{1} {q},\lambda _{2} = \frac{1} {p}\), (3) reduces to (1), while ( 4) reduces to (2). Some other results including multidimensional Hilbert-type integral inequalities are provided by Yang et al. [8], Krnić and Pečarić [9], Yang and Rassias [10, 11], Azar [12], Arpad and Choonghong [13], Kuang and Debnath [14], Zhong [15], Hong [16], Zhong and Yang [17], Yang and Krnić [18], and Li and He [19].

In this chapter, by the use of the methods of weight functions and techniques of real analysis, we give a general multidimensional Hilbert-type integral inequality with a nonhomogeneous kernel and a best possible constant factor. The equivalent forms, the reverses and some Hardy-type inequalities are obtained. Furthermore, we consider the operator expressions with the norm, some particular inequalities with the homogeneous kernel and a large number of particular examples.

2 Some Lemmas

If \(i_{0},j_{0} \in \mathbf{N(N}\) is the set of positive integers), α, β > 0, we put

$$\displaystyle\begin{array}{rcl} & \vert \vert x\vert \vert _{\alpha }:= \left (\sum _{k=1}^{i_{0}}\vert x_{k}\vert ^{\alpha }\right )^{\frac{1} {\alpha } }(x = (x_{1},\ldots,x_{i_{ 0}}) \in \mathbf{R}^{i_{0}}),& {}\\ & \vert \vert y\vert \vert _{\beta }:= \left (\sum _{k=1}^{j_{0}}\vert y_{k}\vert ^{\beta }\right )^{\frac{1} {\beta } }(y = (y_{1},\ldots,y_{j_{ 0}}) \in \mathbf{R}^{j_{0}}).& {}\\ \end{array}$$

Lemma 1.

If \(s \in \mathbf{N,}\gamma,M > 0,\varPsi (u)\) is a nonnegative measurable function in (0,1], and

$$\displaystyle{D_{M}:= \left \{x \in \mathbf{R}_{+}^{s};0 < u =\sum _{ i=1}^{s}\left ( \frac{x_{i}} {M}\right )^{\gamma } \leq 1\right \},}$$

then we have the following expression (cf. [6]):

$$\displaystyle\begin{array}{rcl} & \int \cdots \int _{D_{M}}\varPsi \left (\sum _{i=1}^{s}\left (\frac{x_{i}} {M}\right )^{\gamma }\right )dx_{ 1}\cdots dx_{s}& \\ & = \frac{M^{s}\varGamma ^{s}\big(\frac{1} {\gamma } \big)} {\gamma ^{s}\varGamma \big(\frac{s} {\gamma } \big)} \int _{0}^{1}\varPsi (u)u^{\frac{s} {\gamma } -1}du.&{}\end{array}$$
(5)

In view of (5) and the conditions, it follows that

  1. (i)

    for

    $$\displaystyle{\mathbf{R}_{+}^{s} = \left \{x \in \mathbf{R}_{ +}^{s};0 < u =\sum _{ i=1}^{s}\left ( \frac{x_{i}} {M}\right )^{\gamma } \leq 1(M \rightarrow \infty )\right \}\!\!,}$$

    we have

    $$\displaystyle\begin{array}{rcl} & \int \cdots \int _{\mathbf{R}_{+}^{s}}\varPsi \left (\sum _{i=1}^{s}\left (\frac{x_{i}} {M}\right )^{\gamma }\right )dx_{ 1}\cdots dx_{s} & \\ & =\lim _{M\rightarrow \infty }\frac{M^{s}\varGamma ^{s}\big(\frac{1} {\gamma } \big)} {\gamma ^{s}\varGamma \big(\frac{s} {\gamma } \big)} \int _{0}^{1}\varPsi (u)u^{\frac{s} {\gamma } -1}du;& {}\end{array}$$
    (6)
  2. (ii)

    for

    $$\displaystyle\begin{array}{rcl} \mathbf{\{}x& \in &\mathbf{R}_{+}^{s};\vert \vert x\vert \vert _{\gamma }\geq 1\} {}\\ & =& \left \{x \in \mathbf{R}_{+}^{s}; \frac{1} {M^{\gamma }} < u =\sum _{ i=1}^{s}\left ( \frac{x_{i}} {M}\right )^{\gamma } \leq 1(M \rightarrow \infty )\right \}\!\!, {}\\ \end{array}$$

    setting \(\varPsi (u) = 0\big(u \in \big (0, \frac{1} {M^{\gamma }}\big)\big)\), we have

    $$\displaystyle\begin{array}{rcl} & \int \cdots \int _{\mathbf{\{}x\in \mathbf{R}_{+}^{s};\vert \vert x\vert \vert _{\gamma }\geq 1\}}\varPsi \left (\sum _{i=1}^{s}\left (\frac{x_{i}} {M}\right )^{\gamma }\right )dx_{ 1}\cdots dx_{s}& \\ & =\lim _{M\rightarrow \infty }\frac{M^{s}\varGamma ^{s}\big(\frac{1} {\gamma } \big)} {\gamma ^{s}\varGamma \big(\frac{s} {\gamma } \big)} \int _{ \frac{1} {M^{\gamma }} }^{1}\varPsi (u)u^{\frac{s} {\gamma } -1}du; & {}\end{array}$$
    (7)
  3. (iii)

    for

    $$\displaystyle\begin{array}{rcl} \mathbf{\{}x& \in &\mathbf{R}_{+}^{s};\vert \vert x\vert \vert _{\gamma }\leq 1\} {}\\ & =& \left \{x \in \mathbf{R}_{+}^{s};0 < u =\sum _{ i=1}^{s}\left ( \frac{x_{i}} {M}\right )^{\gamma } \leq \frac{1} {M^{\gamma }}\right \}\!\!, {}\\ \end{array}$$

    setting \(\varPsi (u) = 0\big(u \in \big ( \frac{1} {M^{\gamma }},\infty \big)\big)\), we have

    $$\displaystyle\begin{array}{rcl} & \int \cdots \int _{\mathbf{\{}x\in \mathbf{R}_{+}^{s};\vert \vert x\vert \vert _{\gamma }\leq 1\}}\varPsi \left (\sum _{i=1}^{s}\left (\frac{x_{i}} {M}\right )^{\gamma }\right )dx_{ 1}\cdots dx_{s}& \\ & = \frac{M^{s}\varGamma ^{s}\big(\frac{1} {\gamma } \big)} {\gamma ^{s}\varGamma \big(\frac{s} {\gamma } \big)} \int _{0}^{ \frac{1} {M^{\gamma }} }\varPsi (u)u^{\frac{s} {\gamma } -1}du. & {}\end{array}$$
    (8)

Lemma 2.

For s ∈ N, γ > 0, \(\varepsilon > 0\) , we have

$$\displaystyle\begin{array}{rcl} \int \cdots \int _{\{x\in \mathbf{R}_{+}^{s};\vert \vert x\vert \vert _{\gamma }\geq 1\}}\vert \vert x\vert \vert _{\gamma }^{-s-\varepsilon }dx_{ 1}\cdots dx_{s} = \frac{\varGamma ^{s}\big(\frac{1} {\gamma } \big)} {\varepsilon \gamma ^{s-1}\varGamma \big(\frac{s} {\gamma } \big)},& &{}\end{array}$$
(9)
$$\displaystyle\begin{array}{rcl} \int \cdots \int _{\{x\in \mathbf{R}_{+}^{s};\vert \vert x\vert \vert _{\gamma }\leq 1\}}\vert \vert x\vert \vert _{\gamma }^{-s+\varepsilon }dx_{ 1}\cdots dx_{s} = \frac{\varGamma ^{s}\big(\frac{1} {\gamma } \big)} {\varepsilon \gamma ^{s-1}\varGamma \big(\frac{s} {\gamma } \big)}.& &{}\end{array}$$
(10)

Proof.

By (7), it follows

$$\displaystyle\begin{array}{rcl} & \begin{array}{lll} &&\int \cdots \int _{\{x\in \mathbf{R}_{+}^{s};\vert \vert x\vert \vert _{\gamma }\geq 1\}}\vert \vert x\vert \vert _{\gamma }^{-s-\varepsilon }dx_{1}\cdots dx_{s} \\ && =\int \cdots \int _{\{x\in \mathbf{R}_{+}^{s};\vert \vert x\vert \vert _{\gamma }\geq 1\}}\left \{M\left [\sum _{i=1}^{s}\Big(\frac{x_{i}} {M}\Big)^{\gamma }\right ]^{\frac{1} {\gamma } }\right \}^{-s-\varepsilon }dx_{1}\cdots dx_{s}\end{array} & {}\\ & \begin{array}{lll} && =\lim _{M\rightarrow \infty }\frac{M^{s}\varGamma ^{s}\big(\frac{1} {\gamma } \big)} {\gamma ^{s}\varGamma \big(\frac{s} {\gamma } \big)} \int _{ \frac{1} {M^{\gamma }} }^{1}(Mu^{1/\gamma })^{-s-\varepsilon }u^{\frac{s} {\gamma } -1}du \\ && = \frac{\varGamma ^{s}\big(\frac{1} {\gamma } \big)} {\varepsilon \gamma ^{s-1}\varGamma \big(\frac{s} {\gamma } \big)}.\end{array} & {}\\ \end{array}$$

By (8), we find

$$\displaystyle\begin{array}{rcl} & \begin{array}{ll} &\int \cdots \int _{\{x\in \mathbf{R}_{+}^{s};\vert \vert x\vert \vert _{\gamma }\leq 1\}}\vert \vert x\vert \vert _{\gamma }^{-s+\varepsilon }dx_{1}\cdots dx_{s} \\ & =\int \cdots \int _{\{x\in \mathbf{R}_{+}^{s};\vert \vert x\vert \vert _{\gamma }\leq 1\}}\left \{M\left [\sum _{i=1}^{s}\Big(\frac{x_{i}} {M}\Big)^{\gamma }\right ]^{\frac{1} {\gamma } }\right \}^{-s+\varepsilon }dx_{1}\cdots dx_{s}\end{array} & {}\\ & = \frac{M^{s}\varGamma ^{s}\big(\frac{1} {\gamma } \big)} {\gamma ^{s}\varGamma \big(\frac{s} {\gamma } \big)} \int _{0}^{ \frac{1} {M^{\gamma }} }(Mu^{1/\gamma })^{-s+\varepsilon }u^{\frac{s} {\gamma } -1}du = \frac{\varGamma ^{s}\big(\frac{1} {\gamma } \big)} {\varepsilon \gamma ^{s-1}\varGamma \big(\frac{s} {\gamma } \big)}. & {}\\ \end{array}$$

Hence, we have (9) and (10). The lemma is proved.

Note.

By (9) and (10), for δ = ±1, we have the following unified expression:

$$\displaystyle{ \int \cdots \int _{\{x\in \mathbf{R}_{+}^{s};\vert \vert x\vert \vert _{\gamma }^{\delta }\geq 1\}}\vert \vert x\vert \vert _{\gamma }^{-s-\delta \varepsilon }dx_{ 1}\cdots dx_{s} = \frac{\varGamma ^{s}\big(\frac{1} {\gamma } \big)} {\varepsilon \gamma ^{s-1}\varGamma \big(\frac{s} {\gamma } \big)}. }$$
(11)

Definition 1.

If \(x = (x_{1},\ldots,x_{i_{0}}) \in \mathbf{R}_{+}^{i_{0}},y = (y_{1},\ldots,y_{j_{ 0}}) \in \mathbf{R}_{+}^{j_{0}}\), h(u) is a nonnegative measurable function in R +, σ ∈ R, δ ∈ {−1, 1}, then we define two weight functions ω δ (σ, y) and \(\varpi _{\delta }(\sigma,x)\) as follows:

$$\displaystyle\begin{array}{rcl} \omega _{\delta }(\sigma,y):= \vert \vert y\vert \vert _{\beta }^{\sigma }\int _{\mathbf{R}_{+}^{i_{0}}}h(\vert \vert x\vert \vert _{\alpha }^{\delta }\vert \vert y\vert \vert _{\beta }) \frac{dx} {\vert \vert x\vert \vert _{\alpha }^{i_{0}-\delta \sigma }},& &{}\end{array}$$
(12)
$$\displaystyle\begin{array}{rcl} \varpi _{\delta }(\sigma,x):= \vert \vert x\vert \vert _{\alpha }^{\delta \sigma }\int _{\mathbf{R}_{+}^{j_{0}}}h(\vert \vert x\vert \vert _{\alpha }^{\delta }\vert \vert y\vert \vert _{\beta }) \frac{dy} {\vert \vert y\vert \vert _{\beta }^{j_{0}-\sigma }}.& &{}\end{array}$$
(13)

By (6), we find

$$\displaystyle\begin{array}{rcl} \omega _{\delta }(\sigma,y)& =& \vert \vert y\vert \vert _{\beta }^{\sigma }\int _{\mathbf{R}_{+}^{i_{0}}} \frac{h\Big(M^{\delta }\left [\sum _{i=1}^{i_{0}}\left (\frac{x_{i}} {M}\right )^{\alpha }\right ]^{\frac{\delta }{\alpha } }\vert \vert y\vert \vert _{\beta }\Big)} {M^{i_{0}-\delta \sigma }\left [\sum _{i=1}^{i_{0}}\left (\frac{x_{i}} {M}\right )^{\alpha }\right ]^{\frac{i_{0}-\delta \sigma } {\alpha } }} dx {}\\ & =& \vert \vert y\vert \vert _{\beta }^{\sigma }\lim _{M\rightarrow \infty }\frac{M^{i_{0}}\varGamma ^{i_{0}}\big(\frac{1} {\alpha } \big)} {\alpha ^{i_{0}}\varGamma \big(\frac{i_{0}} {\alpha } \big)} \int _{0}^{1}\frac{h(M^{\delta }u^{\frac{\delta }{\alpha }}\vert \vert y\vert \vert _{\beta })} {M^{i_{0}-\delta \sigma }u^{\frac{i_{0}-\delta \sigma } {\alpha } }} u^{\frac{i_{0}} {\alpha } -1}du {}\\ & =& \vert \vert y\vert \vert _{\beta }^{\sigma }\lim _{M\rightarrow \infty }\frac{M^{\delta \sigma }\varGamma ^{i_{0}}\big(\frac{1} {\alpha } \big)} {\alpha ^{i_{0}}\varGamma \big(\frac{i_{0}} {\alpha } \big)} \int _{0}^{1}h(M^{\delta }u^{\frac{\delta }{\alpha }}\vert \vert y\vert \vert _{\beta })u^{\frac{\delta \sigma }{\alpha }-1}du. {}\\ \end{array}$$

Setting \(v = M^{\delta }u^{\frac{\delta }{\alpha }}\vert \vert y\vert \vert _{ \beta }\) in the above integral, in view of δ = ±1, we obtain

$$\displaystyle{ \omega _{\delta }(\sigma,y) = K_{2}(\sigma ):= \frac{\varGamma ^{i_{0}}\big(\frac{1} {\alpha } \big)} {\alpha ^{i_{0}-1}\varGamma \big(\frac{i_{0}} {\alpha } \big)}k(\sigma ), }$$
(14)

where \(k(\sigma ) =\int _{ 0}^{\infty }h(v)v^{\sigma -1}dv\).

By (6), setting \(v = M\vert \vert x\vert \vert _{\alpha }^{\delta }u^{\frac{1} {\beta } }\), we find

$$\displaystyle\begin{array}{rcl} \varpi _{\delta }(\sigma,x)& =& \vert \vert x\vert \vert _{\alpha }^{\delta \sigma }\int _{\mathbf{R}_{+}^{j_{0}}} \frac{h\left (M\vert \vert x\vert \vert _{\alpha }^{\delta }\left [\sum _{j=1}^{j_{0}}\left (\frac{y_{j}} {M}\right )^{\beta }\right ]^{\frac{1} {\beta } }\right )} {M^{j_{0}-\sigma }\left [\sum _{j=1}^{j_{0}}\left (\frac{y_{j}} {M}\right )^{\beta }\right ]^{\frac{j_{0}-\sigma } {\beta } }} dy \\ & =& \vert \vert x\vert \vert _{\alpha }^{\delta \sigma }\lim _{M\rightarrow \infty }\frac{M^{j_{0}}\varGamma ^{j_{0}}\big(\frac{1} {\beta } \big)} {\beta ^{j_{0}}\varGamma \big(\frac{j_{0}} {\beta } \big)} \int _{0}^{1}\frac{h\left (M\vert \vert x\vert \vert _{\alpha }^{\delta }u^{\frac{1} {\beta } }\right )} {M^{j_{0}-\sigma }u^{\frac{j_{0}-\sigma } {\beta } }} u^{\frac{j_{0}} {\beta } -1}du \\ & =& \vert \vert x\vert \vert _{\alpha }^{\delta \sigma }\lim _{M\rightarrow \infty }\frac{M^{\sigma }\varGamma ^{j_{0}}\big(\frac{1} {\beta } \big)} {\beta ^{j_{0}}\varGamma \big(\frac{j_{0}} {\beta } \big)} \int _{0}^{1}h\left (M\vert \vert x\vert \vert _{\alpha }^{\delta }u^{\frac{1} {\beta } }\right )u^{\frac{\sigma }{\beta }-1}du \\ & =& K_{1}(\sigma ):= \frac{\varGamma ^{j_{0}}\big(\frac{1} {\beta } \big)} {\beta ^{j_{0}-1}\varGamma \big(\frac{j_{0}} {\beta } \big)}k(\sigma ). {}\end{array}$$
(15)

Lemma 3.

As the assumptions of Definition 1 , for k(σ) ∈ R +,\(p \in \mathbf{R}\setminus \{0,1\}, \frac{1} {p} + \frac{1} {q} = 1\) , setting

$$\displaystyle{ \tilde{I}:=\int _{\left \{x\in \mathbf{R}_{+}^{i_{0}};\vert \vert x\vert \vert _{\alpha }^{\delta }\geq 1\right \}}\vert \vert x\vert \vert _{\alpha }^{\delta \sigma -\frac{\delta \varepsilon }{p}-i_{0} }\left [\int _{\left \{y\in \mathbf{R}_{+}^{j_{0}};\vert \vert y\vert \vert _{\beta }\leq 1\right \}}h\left (\vert \vert x\vert \vert _{\alpha }^{\delta }\vert \vert y\vert \vert _{\beta }\right )\vert \vert y\vert \vert _{\beta }^{\sigma + \frac{\varepsilon }{q}-j_{0} }dy\right ]dx, }$$
(16)

then we have

$$\displaystyle{ \varepsilon \tilde{I} \geq \tilde{ K}(\sigma ) + o(1)(\varepsilon \rightarrow 0^{+}), }$$
(17)

where \(\tilde{K}(\sigma ):= L(\alpha,\beta )k(\sigma )\mathbf{,}\)

$$\displaystyle{ L(\alpha,\beta ):= \frac{\varGamma ^{j_{0}}\big(\frac{1} {\beta } \big)} {\beta ^{j_{0}-1}\varGamma \big(\frac{j_{0}} {\beta } \big)} \frac{\varGamma ^{i_{0}}\big(\frac{1} {\alpha } \big)} {\alpha ^{i_{0}-1}\varGamma \big(\frac{i_{0}} {\alpha } \big)}. }$$
(18)

Moreover, if there exists a constant δ 0 > 0, such that for any \(\tilde{\sigma }\in (\sigma -\delta _{0},\sigma +\delta _{0}),k(\tilde{\sigma }) \in \mathbf{R}\), then we have

$$\displaystyle{ \varepsilon \tilde{I} =\tilde{ K}(\sigma ) + o(1)(\varepsilon \rightarrow 0^{+}). }$$
(19)

Proof.

For \(\varepsilon > 0\), setting \(\tilde{\sigma }=\sigma + \frac{\varepsilon }{q}\) and

$$\displaystyle{H(\vert \vert x\vert \vert _{\alpha }^{\delta }):= \vert \vert x\vert \vert _{\alpha }^{\delta \tilde{\sigma }}\int _{\left \{y\in \mathbf{R}_{+}^{j_{0}};\vert \vert y\vert \vert _{\beta }\leq 1\right \}}h\left (\vert \vert x\vert \vert _{\alpha }^{\delta }\vert \vert y\vert \vert _{\beta }\right )\vert \vert y\vert \vert _{\beta }^{\tilde{\sigma }-j_{0} }dy,}$$

in view of (16), it follows

$$\displaystyle{\tilde{I} =\int _{\left \{x\in \mathbf{R}_{+}^{i_{0}};\vert \vert x\vert \vert _{\alpha }^{\delta }\geq 1\right \}}\vert \vert x\vert \vert _{\alpha }^{-\delta \varepsilon -i_{0} }H\left (\vert \vert x\vert \vert _{\alpha }^{\delta }\right )dx.}$$

Putting

$$\displaystyle{\varPsi (u) = h(\vert \vert x\vert \vert _{\alpha }^{\delta }Mu^{\frac{1} {\beta } })M^{\tilde{\sigma }-j_{0}}u^{\frac{1} {\beta } (\tilde{\sigma }-j_{0})},}$$

by (8), we find

$$\displaystyle\begin{array}{rcl} H(\vert \vert x\vert \vert _{\alpha }^{\delta })& =& \vert \vert x\vert \vert _{\beta }^{\delta \tilde{\sigma }}\frac{M^{j_{0}}\varGamma ^{j_{0}}\big(\frac{1} {\beta } \big)} {\beta ^{j_{0}}\varGamma \big(\frac{j_{0}} {\beta } \big)} {}\\ & & \times \int _{0}^{ \frac{1} {M^{\beta }} }h(\vert \vert x\vert \vert _{\alpha }^{\delta }Mu^{\frac{1} {\beta } })M^{\tilde{\sigma }-j_{0}}u^{\frac{1} {\beta } (\tilde{\sigma }-j_{0})}u^{\frac{j_{0}} {\beta } -1}du {}\\ & =& \vert \vert x\vert \vert _{\alpha }^{\delta \tilde{\sigma }}\frac{M^{\tilde{\sigma }}\varGamma ^{j_{0}}\big(\frac{1} {\beta } \big)} {\beta ^{j_{0}}\varGamma \big(\frac{j_{0}} {\beta } \big)} \int _{0}^{ \frac{1} {M^{\beta }} }h(\vert \vert x\vert \vert _{\alpha }^{\delta }Mu^{\frac{1} {\beta } })u^{\frac{\tilde{\sigma }}{\beta }-1}du. {}\\ \end{array}$$

Setting \(v = \vert \vert x\vert \vert _{\alpha }^{\delta }Mu^{\frac{1} {\beta } }\) in the above, it follows

$$\displaystyle{L(\vert \vert x\vert \vert _{\alpha }^{\delta }) = \frac{\varGamma ^{j_{0}}\big(\frac{1} {\beta } \big)} {\beta ^{j_{0}-1}\varGamma \big(\frac{j_{0}} {\beta } \big)}\int _{0}^{\vert \vert x\vert \vert _{\alpha }^{\delta }}h(v)v^{\tilde{\sigma }-1}dv.}$$

Putting \(\varPsi (u) = M^{-\delta \varepsilon -i_{0}}u^{\frac{1} {\alpha } (-\delta \varepsilon -i_{0})}H(M^{\delta }u^{\frac{\delta }{\alpha }})\), for δ = 1, by (7), we obtain

$$\displaystyle\begin{array}{rcl} \tilde{I}& =& \lim _{M\rightarrow \infty }\frac{M^{i_{0}}\varGamma ^{i_{0}}\big(\frac{1} {\alpha } \big)} {\alpha ^{i_{0}}\varGamma \big(\frac{i_{0}} {\alpha } \big)} \int _{ \frac{1} {M^{\alpha }} }^{1}M^{-\varepsilon -i_{0} }u^{\frac{1} {\alpha } (-\varepsilon -i_{0})}H(Mu^{\frac{1} {\alpha } })u^{\frac{i_{0}} {\alpha } -1}du {}\\ & =& \lim _{M\rightarrow \infty }\frac{M^{-\varepsilon }\varGamma ^{i_{0}}\big(\frac{1} {\alpha } \big)} {\alpha ^{i_{0}}\varGamma \big(\frac{i_{0}} {\alpha } \big)} \int _{ \frac{1} {M^{\alpha }} }^{1}H(Mu^{\frac{1} {\alpha } })u^{\frac{-\varepsilon } {\alpha } -1}du {}\\ & =& \frac{\varGamma ^{i_{0}}\big(\frac{1} {\alpha } \big)} {\alpha ^{i_{0}-1}\varGamma \big(\frac{i_{0}} {\alpha } \big)}\int _{1}^{\infty }H(t)t^{-\varepsilon -1}dt(t = Mu^{\frac{1} {\alpha } }); {}\\ \end{array}$$

for δ = −1, by (8), we still find that

$$\displaystyle\begin{array}{rcl} \tilde{I}& =& \frac{M^{i_{0}}\varGamma ^{i_{0}}\big(\frac{1} {\alpha } \big)} {\alpha ^{i_{0}}\varGamma \big(\frac{i_{0}} {\alpha } \big)} \int _{0}^{ \frac{1} {M^{\alpha }} }M^{\varepsilon -i_{0}}u^{\frac{1} {\alpha } (\varepsilon -i_{0})}H(M^{-1}u^{\frac{-1} {\alpha } })u^{\frac{i_{0}} {\alpha } -1}du {}\\ & =& \frac{M^{\varepsilon }\varGamma ^{i_{0}}\big(\frac{1} {\alpha } \big)} {\alpha ^{i_{0}}\varGamma \big(\frac{i_{0}} {\alpha } \big)} \int _{0}^{ \frac{1} {M^{\alpha }} }H(M^{-1}u^{\frac{-1} {\alpha } })u^{\frac{\varepsilon }{\alpha }-1}du {}\\ & =& \frac{\varGamma ^{i_{0}}\big(\frac{1} {\alpha } \big)} {\alpha ^{i_{0}-1}\varGamma \big(\frac{i_{0}} {\alpha } \big)}\int _{1}^{\infty }H(t)t^{-\varepsilon -1}dt(t = M^{-1}u^{\frac{-1} {\alpha } }). {}\\ \end{array}$$

Hence, we find

$$\displaystyle\begin{array}{rcl} \varepsilon \tilde{I}& =& \varepsilon L(\alpha,\beta )\int _{1}^{\infty }t^{-\varepsilon -1}\int _{ 0}^{t}h(v)v^{\tilde{\sigma }-1}dvdt {}\\ & =& \varepsilon L(\alpha,\beta )\left [\int _{1}^{\infty }t^{-\varepsilon -1}\int _{ 0}^{1}h(v)v^{\tilde{\sigma }-1}dvdt\right. {}\\ & & \left.+\int _{1}^{\infty }t^{-\varepsilon -1}\int _{ 1}^{t}h(v)v^{\tilde{\sigma }-1}dvdt\right ] {}\\ \end{array}$$
$$\displaystyle{=\varepsilon L(\alpha,\beta )\left [\frac{1} {\varepsilon } \int _{0}^{1}h(v)v^{\tilde{\sigma }-1}dvdt +\int _{ 1}^{\infty }\left (\int _{ v}^{\infty }t^{-\varepsilon -1}dt\right )h(v)v^{\tilde{\sigma }-1}dv\right ]}$$
$$\displaystyle{ = L(\alpha,\beta )\left [\int _{0}^{1}h(v)v^{\tilde{\sigma }-1}dvdt +\int _{ 1}^{\infty }h(v)v^{\left (\sigma -\frac{\varepsilon }{p}\right )-1}dv\right ]. }$$
(20)

By Fatou lemma (cf. [20]), it follows

$$\displaystyle\begin{array}{rcl} \lim _{\overline{\varepsilon \rightarrow 0^{+}}}\varepsilon \tilde{I}& =& L(\alpha,\beta )\lim _{\overline{\varepsilon \rightarrow 0^{+}}}\left [\int _{0}^{1}h(v)v^{\tilde{\sigma }-1}dvdt +\int _{ 1}^{\infty }h(v)v^{\left (\sigma -\frac{\varepsilon }{p}\right )-1}dv\right ] {}\\ & \geq & L(\alpha,\beta )\left [\int _{0}^{1}\lim _{ \overline{\varepsilon \rightarrow 0^{+}}}h(v)v^{\tilde{\sigma }-1}dvdt\right. {}\\ & & \qquad \qquad \left.+\int _{1}^{\infty }\lim _{ \overline{\varepsilon \rightarrow 0^{+}}}h(v)v^{\left (\sigma -\frac{\varepsilon }{p}\right )-1}dv\right ] = L(\alpha,\beta )k(\sigma ), {}\\ \end{array}$$

and then (17) follows.

Moreover, for \(0 <\varepsilon <\delta _{0}\min \{\vert p\vert,\vert q\vert \},\tilde{\sigma }\in \big (\sigma -\frac{1} {2}\delta _{0},\sigma +\frac{1} {2}\delta _{0}\big)\), since

$$\displaystyle\begin{array}{rcl} h(v)v^{\tilde{\sigma }-1}& \leq & h(v)v^{\left (\sigma -\frac{1} {2} \delta _{0}\right )-1}(v \in (0,1]), {}\\ 0& \leq & \int _{0}^{1}h(v)v^{\left (\sigma -\frac{1} {2} \delta _{0}\right )-1} \leq k\left (\sigma -\frac{1} {2}\delta _{0}\right ) < \infty, {}\\ h(v)v^{\tilde{\sigma }-1}& \leq & h(v)v^{(\sigma +\frac{1} {2} \delta _{0})-1}(v \in [1,\infty )), {}\\ 0& \leq & \int _{1}^{\infty }h(v)v^{(\sigma +\frac{1} {2} \delta _{0})-1} \leq k\left (\sigma +\frac{1} {2}\delta _{0}\right ) < \infty, {}\\ \end{array}$$

by Lebesgue control convergence theorem (cf. [20]), it follows that

$$\displaystyle\begin{array}{rcl} & \int _{0}^{1}h(v)v^{\tilde{\sigma }-1}dv =\int _{ 0}^{1}h(v)v^{\sigma -1}dv + o_{1}(1)(\varepsilon \rightarrow 0^{+}), & {}\\ & \int _{1}^{\infty }h(v)v^{(\sigma -\frac{\varepsilon }{p})-1}dv =\int _{ 1}^{\infty }h(v)v^{\sigma -1}dv + o_{ 2}(1)(\varepsilon \rightarrow 0^{+}).& {}\\ \end{array}$$

Then by (20), (19) follows. The lemma is proved.

Lemma 4.

As the assumptions of Definition 1 , if \(p \in \mathbf{R}\setminus \{0,1\}, \frac{1} {p} + \frac{1} {q} = 1\),\(f(x) = f(x_{1},\ldots,x_{i_{0}}) \geq 0\),\(g(y) = g(y_{1},\ldots,y_{j_{0}}) \geq 0\) , then

  1. (i)

    for p > 1, we have the following inequality:

    $$\displaystyle\begin{array}{rcl} J_{1}&:=& \left \{\int _{\mathbf{R}_{+}^{j_{0}}} \frac{\vert \vert y\vert \vert _{\beta }^{p\sigma -j_{0}}} {\left [\omega _{\delta }(\sigma,y)\right ]^{p-1}}\left (\int _{\mathbf{R}_{+}^{i_{0}}}h(\vert \vert x\vert \vert _{\alpha }^{\delta }\vert \vert y\vert \vert _{\beta })f(x)dx\right )^{p}dy\right \}^{\frac{1} {p} } \\ & \leq &\left \{\int _{\mathbf{R}_{+}^{i_{0}}}\varpi _{\delta }(\sigma,x)\vert \vert x\vert \vert _{\alpha }^{p\left (i_{0}-\delta \sigma \right )-i_{0} }f^{p}(x)dx\right \}^{\frac{1} {p} }; {}\end{array}$$
    (21)
  2. (ii)

    for 0 < p < 1, or p < 0, we have the reverse of (21) .

Proof.

  1. (i)

    For p > 1, by Hölder’s inequality with weight (cf. [21]), it follows

    $$\displaystyle\begin{array}{rcl} & & \int _{\mathbf{R}_{+}^{i_{0}}}h(\vert \vert x\vert \vert _{\alpha }^{\delta }\vert \vert y\vert \vert _{\beta })f(x)dx \\ & & \quad =\int _{\mathbf{R}_{+}^{i_{0}}}h(\vert \vert x\vert \vert _{\alpha }^{\delta }\vert \vert y\vert \vert _{\beta })\left [\frac{\vert \vert x\vert \vert _{\alpha }^{(i_{0}-\delta \sigma )/q}f(x)} {\vert \vert y\vert \vert _{\beta }^{(j_{0}-\sigma )/p}} \right ]\left [\frac{\vert \vert y\vert \vert _{\beta }^{(j_{0}-\sigma )/p}} {\vert \vert x\vert \vert _{\alpha }^{(i_{0}-\delta \sigma )/q}}\right ]dx \\ & & \quad \leq \left \{\int _{\mathbf{R}_{+}^{i_{0}}}h(\vert \vert x\vert \vert _{\alpha }^{\delta }\vert \vert y\vert \vert _{\beta })\frac{\vert \vert x\vert \vert _{\alpha }^{(i_{0}-\delta \sigma )(p-1)}} {\vert \vert y\vert \vert _{\beta }^{j_{0}-\sigma }} f^{p}(x)dx\right \}^{\frac{1} {p} } \\ & & \qquad \times \left \{\int _{\mathbf{R}_{+}^{i_{0}}}h(\vert \vert x\vert \vert _{\alpha }^{\delta }\vert \vert y\vert \vert _{\beta })\frac{\vert \vert y\vert \vert _{\beta }^{(j_{0}-\sigma )(q-1)}} {\vert \vert x\vert \vert _{\alpha }^{i_{0}-\delta \sigma }} dx\right \}^{\frac{1} {q} } \\ & & \quad = [\omega _{\delta }(\sigma,y)]^{\frac{1} {q} }\vert \vert y\vert \vert _{\beta }^{\frac{j_{0}} {p} -\sigma } \\ &&\qquad \times \left \{\int _{\mathbf{R}_{+}^{i_{0}}}h(\vert \vert x\vert \vert _{\alpha }^{\delta }\vert \vert y\vert \vert _{\beta })\frac{\vert \vert x\vert \vert _{\alpha }^{(i_{0}-\delta \sigma )(p-1)}} {\vert \vert y\vert \vert _{\beta }^{j_{0}-\sigma }} f^{p}(x)dx\right \}^{\frac{1} {p} }. {}\end{array}$$
    (22)

    Then by Fubini theorem (cf. [20]), we have

    $$\displaystyle\begin{array}{rcl} J_{1}& \leq &\left \{\int _{\mathbf{R}_{+}^{j_{0}}}\left [\int _{\mathbf{R}_{+}^{i_{0}}}h(\vert \vert x\vert \vert _{\alpha }^{\delta }\vert \vert y\vert \vert _{\beta })\frac{\vert \vert x\vert \vert _{\alpha }^{(i_{0}-\delta \sigma )(p-1)}} {\vert \vert y\vert \vert _{\beta }^{j_{0}-\sigma }} f^{p}(x)dx\right ]dy\right \}^{\frac{1} {p} } \\ & =& \left \{\int _{\mathbf{R}_{+}^{i_{0}}}\left [\int _{\mathbf{R}_{+}^{j_{0}}}h(\vert \vert x\vert \vert _{\alpha }\vert \vert y\vert \vert _{\beta })\frac{\vert \vert x\vert \vert _{\alpha }^{(i_{0}-\delta \sigma )(p-1)}} {\vert \vert y\vert \vert _{\beta }^{j_{0}-\sigma }} dy\right ]f^{p}(x)dx\right \}^{\frac{1} {p} } \\ & =& \left \{\int _{\mathbf{R}_{+}^{i_{0}}}\varpi _{\delta }(\sigma,x)\vert \vert x\vert \vert _{\alpha }^{p(i_{0}-\delta \sigma )-i_{0} }f^{p}(x)dx\right \}^{\frac{1} {p} }. {}\end{array}$$
    (23)

    Hence, (21) follows.

  2. (ii)

    For 0 < p < 1, or p < 0, by the reverse Hölder’s inequality with weight (cf. [21]), we obtain the reverse of (22). Then by Fubini theorem, we still can obtain the reverse of (21). The lemma is proved.

Lemma 5.

As the assumptions of Lemma 4 , then

  1. (i)

    for p > 1, we have the following inequality equivalent to (21) :

    $$\displaystyle\begin{array}{rcl} I&:=& \int _{\mathbf{R}_{+}^{j_{0}}}\int _{\mathbf{R}_{+}^{i_{0}}}h(\vert \vert x\vert \vert _{\alpha }^{\delta }\vert \vert y\vert \vert _{\beta })f(x)g(y)dxdy \\ & \leq &\left \{\int _{\mathbf{R}_{+}^{i_{0}}}\varpi _{\delta }(\sigma,x)\vert \vert x\vert \vert _{\alpha }^{p(i_{0}-\delta \sigma )-i_{0} }f^{p}(x)dx\right \}^{\frac{1} {p} } \\ & & \times \left \{\int _{\mathbf{R}_{+}^{j_{0}}}\omega _{\delta }(\sigma,y)\vert \vert y\vert \vert _{\beta }^{q(j_{0}-\sigma )-j_{0} }g^{q}(y)dy\right \}^{\frac{1} {q} }; {}\end{array}$$
    (24)
  2. (ii)

    for 0 < p < 1, or p < 0, we have the reverse of (24) equivalent to the reverse of (21) .

Proof.

  1. (i)

    For p > 1, by Hölder’s inequality (cf. [21]), it follows

    $$\displaystyle\begin{array}{rcl} I& =& \int _{\mathbf{R}_{+}^{j_{0}}} \frac{\vert \vert y\vert \vert _{\beta }^{\frac{j_{0}} {q} -(j_{0}-\sigma )}} {\left [\omega _{\delta }(\sigma,y)\right ]^{\frac{1} {q} }} \left [\int _{\mathbf{R}_{+}^{i_{0}}}h(\vert \vert x\vert \vert _{\alpha }^{\delta }\vert \vert y\vert \vert _{\beta })f(x)dx\right ] {}\\ & & \times \left [[\omega _{\delta }(\sigma,y)]^{\frac{1} {q} }\vert \vert y\vert \vert _{\beta }^{(j_{0}-\sigma )-\frac{j_{0}} {q} }g(y)\right ]dy {}\\ \end{array}$$
    $$\displaystyle{ \leq J_{1}\left \{\int _{\mathbf{R}_{+}^{j_{0}}}\omega _{\delta }(\sigma,y)\vert \vert y\vert \vert _{\beta }^{q(j_{0}-\sigma )-j_{0} }g^{q}(y)dy\right \}^{\frac{1} {q} }. }$$
    (25)

    Then by (21), we have (24).

    On the other hand, assuming that (24) is valid, we set

    $$\displaystyle{g(y):= \frac{\vert \vert y\vert \vert _{\beta }^{p\sigma -j_{0}}} {[\omega _{\delta }(\sigma,y)]^{p-1}}\left (\int _{\mathbf{R}_{+}^{i_{0}}}h(\vert \vert x\vert \vert _{\alpha }^{\delta }\vert \vert y\vert \vert _{\beta })f(x)dx\right )^{p-1},y \in \mathbf{R}_{ +}^{j_{0} }.}$$

    Then it follows

    $$\displaystyle{J_{1}^{p} =\int _{\mathbf{ R}_{+}^{j_{0}}}\omega _{\delta }(\sigma,y)\vert \vert y\vert \vert _{\beta }^{q(j_{0}-\sigma )-j_{0} }g^{q}(y)dy.}$$

    If J 1 = 0, then (21) is trivially valid; if J 1 = , then by (23), (21) keeps the form of equality ( = ). Suppose that 0 < J 1 < . By (24), we have

    $$\displaystyle\begin{array}{rcl} 0& <& \int _{\mathbf{R}_{+}^{j_{0}}}\omega _{\delta }(\sigma,y)\vert \vert y\vert \vert _{\beta }^{q(j_{0}-\sigma )-j_{0} }g^{q}(y)dy = J_{ 1}^{p} = I {}\\ & \leq &\left \{\int _{\mathbf{R}_{+}^{i_{0}}}\varpi _{\delta }(\sigma,x)\vert \vert x\vert \vert _{\alpha }^{p(i_{0}-\delta \sigma )-i_{0} }f^{p}(x)dx\right \}^{\frac{1} {p} } {}\\ & & \times \left \{\int _{\mathbf{R}_{+}^{j_{0}}}\omega _{\delta }(\sigma,y)\vert \vert y\vert \vert _{\beta }^{q(j_{0}-\sigma )-j_{0} }g^{q}(y)dy\right \}^{\frac{1} {q} } < \infty. {}\\ \end{array}$$

    It follows

    $$\displaystyle\begin{array}{rcl} J_{1}& =& \left \{\int _{\mathbf{R}_{+}^{j_{0}}}\omega _{\delta }(\sigma,y)\vert \vert y\vert \vert _{\beta }^{q(j_{0}-\sigma )-j_{0} }g^{q}(y)dy\right \}^{\frac{1} {p} } {}\\ & \leq &\left \{\int _{\mathbf{R}_{+}^{i_{0}}}\varpi _{\delta }(\sigma,x)\vert \vert x\vert \vert _{\alpha }^{p(i_{0}-\delta \sigma )-i_{0} }f^{p}(x)dx\right \}^{\frac{1} {p} }, {}\\ \end{array}$$

    and then (21) follows. Hence, (21) and (24) are equivalent.

  2. (ii)

    For 0 < p < 1, or p < 0, by the same way, we can obtain the reverse of (24) equivalent to the reverse of (21). The lemma is proved.

3 Main Results and Operator Expressions

Setting

$$\displaystyle\begin{array}{rcl} \varPhi _{\delta }(x)&:=& \vert \vert x\vert \vert _{\alpha }^{p(i_{0}-\delta \sigma )-i_{0} }, {}\\ \varPsi (y)&:=& \vert \vert y\vert \vert _{\beta }^{q(j_{0}-\sigma )-j_{0} }(x \in \mathbf{R}_{+}^{i_{0} },y \in \mathbf{R}_{+}^{j_{0} }), {}\\ \end{array}$$

by Lemmas 3– 5, it follows

Theorem 1.

Suppose that α,β > 0, σ ∈ R ,h(v) ≥ 0,

$$\displaystyle\begin{array}{rcl} & k(\sigma ) =\int _{ 0}^{\infty }h(v)v^{\sigma -1}dv \in \mathbf{R}_{+}, & {}\\ & K(\sigma ):= \left [ \frac{\varGamma ^{j_{0}}\big(\frac{1} {\beta } \big)} {\beta ^{j_{0}-1}\varGamma \big(\frac{j_{0}} {\beta } \big)}\right ]^{\frac{1} {p} }\left [ \frac{\varGamma ^{i_{0}}\big(\frac{1} {\alpha } \big)} {\alpha ^{i_{0}-1}\varGamma \left (\frac{i_{0}} {\alpha } \right )}\right ]^{\frac{1} {q} }k(\sigma ),& {}\\ \end{array}$$

\(\delta \in \{-1,1\},p \in \mathbf{R}\setminus \{0,1\}\),\(\frac{1} {p} + \frac{1} {q} = 1\),\(\ f(x) = f(x_{1},\ldots,x_{i_{0}}) \geq 0\),\(g(y) = g(y_{1},\ldots,y_{j_{0}})\) ≥ 0,

$$\displaystyle\begin{array}{rcl} 0& <& \vert \vert f\vert \vert _{p,\varPhi _{\delta }} = \left \{\int _{\mathbf{R}_{+}^{i_{0}}}\varPhi _{\delta }(x)f^{p}(x)dx\right \}^{\frac{1} {p} } < \infty, {}\\ 0& <& \vert \vert g\vert \vert _{q,\varPsi } = \left \{\int _{\mathbf{R}_{+}^{j_{0}}}\varPsi (y)g^{q}(y)dy\right \}^{\frac{1} {q} } < \infty. {}\\ \end{array}$$
  1. (i)

    If p > 1, then we have the following equivalent inequalities with the best possible constant factor K(σ):

    $$\displaystyle{ I =\int _{\mathbf{R}_{+}^{j_{0}}}\int _{\mathbf{R}_{+}^{i_{0}}}h(\vert \vert x\vert \vert _{\alpha }^{\delta }\vert \vert y\vert \vert _{\beta })f(x)g(y)dxdy < K(\sigma )\vert \vert f\vert \vert _{p,\varPhi _{\delta }}\vert \vert g\vert \vert _{q,\varPsi }, }$$
    (26)
    $$\displaystyle\begin{array}{rcl} J&:=& \left \{\int _{\mathbf{R}_{+}^{j_{0}}}\vert \vert y\vert \vert _{\beta }^{p\sigma -j_{0} }\left (\int _{\mathbf{R}_{+}^{i_{0}}}h(\vert \vert x\vert \vert _{\alpha }^{\delta }\vert \vert y\vert \vert _{\beta })f(x)dx\right )^{p}dy\right \}^{\frac{1} {p} } \\ & & < K(\sigma )\vert \vert f\vert \vert _{p,\varPhi _{\delta }}; {}\end{array}$$
    (27)
  2. (ii)

    if 0 < p < 1, or p < 0, there exists a constant δ 0 > 0, such that for any \(\tilde{\sigma }\in (\sigma -\delta _{0},\sigma +\delta _{0})\),\(k(\tilde{\sigma }) \in \mathbf{R}\) , then we still have the equivalent reverses of (26) and (27) with the same best constant factor K(σ).

Proof.

  1. (i)

    For p > 1, by the conditions, we can prove that (22) takes the form of strict inequality for a.e. \(y \in \mathbf{R}_{+}^{j_{0}}\). Otherwise, if (22) takes the form of equality for a \(y \in \mathbf{R}_{+}^{j_{0}}\), then there exist constants A and B, which are not all zero, such that

    $$\displaystyle{ A\frac{\vert \vert x\vert \vert _{\alpha }^{(i_{0}-\delta \sigma )(p-1)}} {\vert \vert y\vert \vert _{\beta }^{j_{0}-\sigma }} f^{p}(x) = B\frac{\vert \vert y\vert \vert _{\beta }^{(j_{0}-\sigma )(q-1)}} {\vert \vert x\vert \vert _{\alpha }^{i_{0}-\delta \sigma }} \mathrm{\,\, a.e.\,\, in \,\,}x \in \mathbf{R}_{+}^{i_{0} }. }$$
    (28)

    If A = 0, then B = 0, which is impossible; if A ≠ 0, then (28) reduces to

    $$\displaystyle{\vert \vert x\vert \vert _{\alpha }^{p(i_{0}-\delta \sigma )-i_{0} }f^{p}(x) = \frac{B\vert \vert y\vert \vert _{\beta }^{q(j_{0}-\sigma )}} {A\vert \vert x\vert \vert _{\alpha }^{i_{0}}} \mathrm{\,\,a.e.\,\, in \,\,}x \in \mathbf{R}_{+}^{i_{0} },}$$

    which contradicts the fact that \(0 < \vert \vert f\vert \vert _{p,\varPhi _{\delta }} < \infty \). In fact, by (9) (for \(\varepsilon \rightarrow 0^{+})\), it follows

    $$\displaystyle{\int _{\mathbf{R}_{+}^{i_{0}}}\vert \vert x\vert \vert _{\alpha }^{-i_{0} }dx \geq \int _{\left \{x\in \mathbf{R}_{+}^{i_{0}};\vert \vert x\vert \vert _{\alpha }\geq 1\right \}}\vert \vert x\vert \vert _{\alpha }^{-i_{0} }dx = \infty.}$$

    Hence (22) still takes the form of strict inequality. By (14) and (15), we obtain (27).

    Similarly to (25), we still have

    $$\displaystyle{ I \leq J\left \{\int _{\mathbf{R}_{+}^{j_{0}}}\vert \vert y\vert \vert _{\beta }^{q(j_{0}-\sigma )-j_{0} }g^{q}(y)dy\right \}^{\frac{1} {q} }. }$$
    (29)

    Then by (29) and (27), we have (26). It is evident that by Lemma 5 and the assumptions, inequalities (27) and ( 26) are also equivalent.

    For \(\varepsilon > 0\), we set \(\tilde{f}(x),\tilde{g}(y)\) as follows:

    $$\displaystyle\begin{array}{rcl} & \tilde{f}(x):= \left \{\begin{array}{l} 0,\quad 0 < \vert \vert x\vert \vert _{\alpha }^{\delta } < 1, \\ \vert \vert x\vert \vert _{\alpha }^{\delta \big(\sigma -\frac{\varepsilon }{p}\big)-i_{0} },\quad \vert \vert x\vert \vert _{\alpha }^{\delta }\geq 1,\end{array} \right.& {}\\ & \tilde{g}(y):= \left \{\begin{array}{l} \vert \vert y\vert \vert _{\beta }^{\sigma + \frac{\varepsilon }{q}-j_{0} },\quad 0 < \vert \vert y\vert \vert _{\beta }\leq 1, \\ 0,\quad \vert \vert y\vert \vert _{\beta }\geq 1.\end{array} \right.& {}\\ \end{array}$$

    In view of (11) and (10), it follows

    $$\displaystyle\begin{array}{rcl} & & \vert \vert \tilde{f}\vert \vert _{p,\varPhi _{\delta }}\vert \vert \tilde{g}\vert \vert _{q,\varPsi } {}\\ & & \quad = \left \{\int _{\left \{x\in \mathbf{R}_{+}^{i_{0}};\vert \vert x\vert \vert _{\alpha }^{\delta }\geq 1\right \}}\vert \vert x\vert \vert _{\alpha }^{-i_{0}-\delta \varepsilon }dx\right \}^{\frac{1} {p} }\left \{\int _{ \left \{y\in \mathbf{R}_{+}^{j_{0}};\vert \vert y\vert \vert _{\beta }\leq 1\right \}}\vert \vert y\vert \vert _{\beta }^{-j_{0}+\varepsilon }dy\right \}^{\frac{1} {q} } {}\\ & & \quad = \frac{1} {\varepsilon } \left \{ \frac{\varGamma ^{i_{0}}\left (\frac{1} {\alpha } \right )} {\alpha ^{i_{0}-1}\varGamma \left (\frac{i_{0}} {\alpha } \right )}\right \}^{\frac{1} {p} }\left \{ \frac{\varGamma ^{j_{0}}\left (\frac{1} {\beta } \right )} {\beta ^{j_{0}-1}\varGamma \left (\frac{j_{0}} {\beta } \right )}\right \}^{\frac{1} {q} }. {}\\ \end{array}$$

    If there exists a constant K ≤ K(σ), such that (26) is valid when replacing K(σ) by K, then in particular, by (16) and ( 17), we have

    $$\displaystyle\begin{array}{rcl} & & \tilde{K}(\sigma ) + o(1) \leq \varepsilon \tilde{ I} {}\\ & & \quad =\varepsilon \int _{\mathbf{R}_{+}^{j_{0}}}\int _{\mathbf{R}_{+}^{i_{0}}}h(\vert \vert x\vert \vert _{\alpha }^{\delta }\vert \vert y\vert \vert _{\beta })\tilde{f}(x)\tilde{g}(y)dxdy {}\\ & & \quad <\varepsilon K\vert \vert \tilde{f}\vert \vert _{p,\varPhi _{\delta }}\vert \vert \tilde{g}\vert \vert _{q,\varPsi } {}\\ & & \quad = K\left \{ \frac{\varGamma ^{i_{0}}\left (\frac{1} {\alpha } \right )} {\alpha ^{i_{0}-1}\varGamma \left (\frac{i_{0}} {\alpha } \right )}\right \}^{\frac{1} {p} }\left \{ \frac{\varGamma ^{j_{0}}\left (\frac{1} {\beta } \right )} {\beta ^{j_{0}-1}\varGamma \left (\frac{j_{0}} {\beta } \right )}\right \}^{\frac{1} {q} }, {}\\ \end{array}$$

    and then we find \(K(\sigma ) \leq K(\varepsilon \rightarrow 0^{+})\). Hence K = K(σ) is the best possible constant factor of (26).

    By the equivalency, we can prove that the constant factor K(σ) in (27) is the best possible. Otherwise, we would reach a contradiction by (29) that the constant factor K(σ) in (26) is not the best possible.

  2. (ii)

    For 0 < p < 1, or p < 0, by the same way, we still can obtain the equivalent reverses of (26) and (27). For \(\varepsilon > 0\), we set \(\tilde{f}(x),\tilde{g}(y)\) as the case of p > 1. If there exists a constant K ≥ K(σ), such that the reverse of (26) is valid when replacing K(σ) by K, then in particular, by (16) and (19), we have

    $$\displaystyle\begin{array}{rcl} & & \tilde{K}(\sigma ) + o(1) =\varepsilon \tilde{ I} {}\\ & & \quad =\varepsilon \int _{\mathbf{R}_{+}^{j_{0}}}\int _{\mathbf{R}_{+}^{i_{0}}}h(\vert \vert x\vert \vert _{\alpha }^{\delta }\vert \vert y\vert \vert _{\beta })\tilde{f}(x)\tilde{g}(y)dxdy {}\\ & & \quad >\varepsilon K\vert \vert \tilde{f}\vert \vert _{p,\varPhi _{\delta }}\vert \vert \tilde{g}\vert \vert _{q,\varPsi } {}\\ & & \quad = K\left \{ \frac{\varGamma ^{i_{0}}\big(\frac{1} {\alpha } \big)} {\alpha ^{i_{0}-1}\varGamma \left (\frac{i_{0}} {\alpha } \right )}\right \}^{\frac{1} {p} }\left \{ \frac{\varGamma ^{j_{0}}\left (\frac{1} {\beta } \right )} {\beta ^{j_{0}-1}\varGamma \left (\frac{j_{0}} {\beta } \right )}\right \}^{\frac{1} {q} }, {}\\ \end{array}$$

    and then we find \(K(\sigma ) \geq K(\varepsilon \rightarrow 0^{+})\). Hence K = K(σ) is the best possible constant factor of the reverse of (26). By the equivalency, we can prove that the constant factor K(σ) in the reverse of (27) is the best possible. Otherwise, we would reach a contradiction by the reverse of (29) that the constant factor K(σ) in the reverse of (26) is not the best possible. The theorem is proved.

In particular, for δ = 1 in Theorem 1, we have

Corollary 1.

Suppose that α,β > 0, σ ∈ R ,h(v) ≥ 0,

$$\displaystyle\begin{array}{rcl} & k(\sigma ) =\int _{ 0}^{\infty }h(v)v^{\sigma -1}dv \in \mathbf{R}_{+}, & {}\\ & K(\sigma ) = \left [ \frac{\varGamma ^{j_{0}}\big(\frac{1} {\beta } \big)} {\beta ^{j_{0}-1}\varGamma \big(\frac{j_{0}} {\beta } \big)}\right ]^{\frac{1} {p} }\left [ \frac{\varGamma ^{i_{0}}\big(\frac{1} {\alpha } \big)} {\alpha ^{i_{0}-1}\varGamma \big(\frac{i_{0}} {\alpha } \big)}\right ]^{\frac{1} {q} }k(\sigma ),& {}\\ \end{array}$$

\(p \in \mathbf{R}\setminus \{0,1\}\),\(\frac{1} {p} + \frac{1} {q} = 1\),\(\ f(x) = f(x_{1},\ldots,x_{i_{0}}) \geq 0\),\(g(y) = g(y_{1},\ldots,y_{j_{0}}) \geq 0\),

$$\displaystyle\begin{array}{rcl} 0& <& \vert \vert f\vert \vert _{p,\varPhi _{1}} = \left \{\int _{\mathbf{R}_{+}^{i_{0}}}\varPhi _{1}(x)f^{p}(x)dx\right \}^{\frac{1} {p} } < \infty, {}\\ 0& <& \vert \vert g\vert \vert _{q,\varPsi } = \left \{\int _{\mathbf{R}_{+}^{j_{0}}}\varPsi (y)g^{q}(y)dy\right \}^{\frac{1} {q} } < \infty. {}\\ \end{array}$$
  1. (i)

    If p > 1, then we have the following equivalent inequalities with the best possible constant factor K(σ):

    $$\displaystyle{ I =\int _{\mathbf{R}_{+}^{j_{0}}}\int _{\mathbf{R}_{+}^{i_{0}}}h(\vert \vert x\vert \vert _{\alpha }\vert \vert y\vert \vert _{\beta })f(x)g(y)dxdy < K(\sigma )\vert \vert f\vert \vert _{p,\varPhi _{1}}\vert \vert g\vert \vert _{q,\varPsi }, }$$
    (30)
    $$\displaystyle\begin{array}{rcl} J&:=& \left \{\int _{\mathbf{R}_{+}^{j_{0}}}\vert \vert y\vert \vert _{\beta }^{p\sigma -j_{0} }\left (\int _{\mathbf{R}_{+}^{i_{0}}}h(\vert \vert x\vert \vert _{\alpha }\vert \vert y\vert \vert _{\beta })f(x)dx\right )^{p}dy\right \}^{\frac{1} {p} } \\ & & < K(\sigma )\vert \vert f\vert \vert _{p,\varPhi _{1}}; {}\end{array}$$
    (31)
  2. (ii)

    if 0 < p < 1, or p < 0, there exists a constant δ 0 > 0, such that for any \(\tilde{\sigma }\in (\sigma -\delta _{0},\sigma +\delta _{0})\),\(k(\tilde{\sigma }) \in \mathbf{R}\) , then we still have the equivalent reverses of (30) and (31) with the same best constant factor K(σ).

For i 0 = j 0 = α = β = 1 in Corollary 1, we have

Corollary 2.

Assuming that \(\sigma \in \mathbf{R},k(\sigma ) \in \mathbf{R}_{+},p \in \mathbf{R}\setminus \{0,1\}, \frac{1} {p} + \frac{1} {q} = 1\) , we set

$$\displaystyle{\varphi (x):= x^{p(1-\sigma )-1},\psi (y):= y^{q(1-\sigma )-1}(x,y > 0).}$$

If f(x) ≥ 0, g(y) ≥ 0,

$$\displaystyle\begin{array}{rcl} 0& <& \vert \vert f\vert \vert _{p,\varphi } = \left \{\int _{0}^{\infty }\varphi (x)f^{p}(x)dx\right \}^{\frac{1} {p} } < \infty, {}\\ 0& <& \vert \vert g\vert \vert _{q,\psi } = \left \{\int _{0}^{\infty }\psi (y)g^{q}(y)dy\right \}^{\frac{1} {q} } < \infty, {}\\ \end{array}$$

then (i) for p > 1, we have the following equivalent inequalities with the best possible constant factor k(σ):

$$\displaystyle\begin{array}{rcl} \int _{0}^{\infty }\int _{ 0}^{\infty }h(xy)f(x)g(y)dxdy < k(\sigma )\vert \vert f\vert \vert _{ p,\varphi }\vert \vert g\vert \vert _{q,\psi },& &{}\end{array}$$
(32)
$$\displaystyle\begin{array}{rcl} \left \{\int _{0}^{\infty }y^{p\sigma -1}\left [\int _{ 0}^{\infty }h(xy)f(x)dx\right ]^{p}dy\right \}^{\frac{1} {p} } < k(\sigma )\vert \vert f\vert \vert _{p,\varphi };& & {}\end{array}$$
(33)

(ii) for 0 < p < 1, or p < 0, there exists a constant δ 0 > 0, such that for any \(\tilde{\sigma }\in (\sigma -\delta _{0},\sigma +\delta _{0})\),\(k(\tilde{\sigma }) \in \mathbf{R}\) , we have the equivalent reverses of (32) and (33) with the same best constant factor.

As the assumptions of Theorem 1, for p > 1, in view of \(J < K(\sigma )\vert \vert f\vert \vert _{\varPhi _{\delta }}\), we can give the following definition:

Definition 2.

Define a multidimensional Hilbert-type integral operator

$$\displaystyle{ T: \mathbf{L}_{p,\varPhi _{\delta }}(\mathbf{R}_{+}^{i_{0} }) \rightarrow \mathbf{L}_{p,\varPsi ^{1-p}}(\mathbf{R}_{+}^{j_{0} }) }$$
(34)

as follows: For \(f \in \mathbf{L}_{p,\varPhi _{\delta }}(\mathbf{R}_{+}^{i_{0}}),\) there exists a unique representation

$$\displaystyle{Tf \in \mathbf{L}_{p,\varPsi ^{1-p}}(\mathbf{R}_{+}^{j_{0} }),}$$

satisfying

$$\displaystyle{ (Tf)(y):=\int _{\mathbf{R}_{+}^{i_{0}}}h(\vert \vert x\vert \vert _{\alpha }^{\delta }\vert \vert y\vert \vert _{\beta })f(x)dx(y \in \mathbf{R}_{+}^{j_{0} }). }$$
(35)

For \(g \in \mathbf{L}_{q,\varPsi }(\mathbf{R}_{+}^{j_{0}})\), we define the following formal inner product of Tf and g as follows:

$$\displaystyle{ (Tf,g):=\int _{\mathbf{R}_{+}^{j_{0}}}\int _{\mathbf{R}_{+}^{i_{0}}}h(\vert \vert x\vert \vert _{\alpha }^{\delta }\vert \vert y\vert \vert _{\beta })f(x)g(y)dxdy. }$$
(36)

Then by Theorem 1, for \(p > 1,0 < \vert \vert f\vert \vert _{p,\varPhi _{\delta }},\vert \vert g\vert \vert _{q,\varPsi } < \infty \), we have the following equivalent inequalities:

$$\displaystyle\begin{array}{rcl} (Tf,g) < K(\sigma )\vert \vert f\vert \vert _{p,\varPhi _{\delta }}\vert \vert g\vert \vert _{q,\varPsi },& &{}\end{array}$$
(37)
$$\displaystyle\begin{array}{rcl} \vert \vert Tf\vert \vert _{p,\varPsi ^{1-p}} < K(\sigma )\vert \vert f\vert \vert _{p,\varPhi _{\delta }}.& &{}\end{array}$$
(38)

It follows that T is bounded with

$$\displaystyle{\vert \vert T\vert \vert:=\sup _{f(\neq \theta )\in \mathbf{L}_{p,\varPhi _{ \delta }}(\mathbf{R}_{+}^{i_{0}})}\frac{\vert \vert Tf\vert \vert _{p,\varPsi ^{1-p}}} {\vert \vert f\vert \vert _{p,\varPhi _{\delta }}} \leq K(\sigma ).}$$

Since the constant factor K(σ) in (38) is the best possible, we have

$$\displaystyle\begin{array}{rcl} \vert \vert T\vert \vert = K(\sigma )& =& \left [ \frac{\varGamma ^{j_{0}}\big(\frac{1} {\beta } \big)} {\beta ^{j_{0}-1}\varGamma \big(\frac{j_{0}} {\beta } \big)}\right ]^{\frac{1} {p} } \\ & & \times \left [ \frac{\varGamma ^{i_{0}}\big(\frac{1} {\alpha } \big)} {\alpha ^{i_{0}-1}\varGamma \big(\frac{i_{0}} {\alpha } \big)}\right ]^{ \frac{1} {,q} }k(\sigma ).{}\end{array}$$
(39)

4 A Corollary for δ = −1

Corollary 3.

Suppose that α,β > 0, μ,σ ∈ R ,μ + σ = λ,k λ (x,y) ≥ 0 is a homogeneous function of degree −λ,

$$\displaystyle\begin{array}{rcl} & k_{\lambda }(\sigma ):=\int _{ 0}^{\infty }k_{\lambda }(1,v)v^{\sigma -1}dv \in \mathbf{R}_{+}, & {}\\ & K_{\lambda }(\sigma ):= \left [ \frac{\varGamma ^{j_{0}}\big(\frac{1} {\beta } \big)} {\beta ^{j_{0}-1}\varGamma \big(\frac{j_{0}} {\beta } \big)}\right ]^{\frac{1} {p} }\left [ \frac{\varGamma ^{i_{0}}\big(\frac{1} {\alpha } \big)} {\alpha ^{i_{0}-1}\varGamma \big(\frac{i_{0}} {\alpha } \big)}\right ]^{\frac{1} {q} }k_{\lambda }(\sigma ),& {}\\ \end{array}$$

\(p \in \mathbf{R}\setminus \{0,1\}\),\(\frac{1} {p} + \frac{1} {q} = 1,\varPhi (x):= x^{p(i_{0}-\mu )-i_{0}},F(x) = F(x_{ 1},\ldots,x_{i_{0}}) \geq 0\),\(g(y) = g(y_{1},\ldots,y_{j_{0}}) \geq 0\),

$$\displaystyle\begin{array}{rcl} 0& <& \vert \vert F\vert \vert _{p,\varPhi } = \left \{\int _{\mathbf{R}_{+}^{i_{0}}}\varPhi (x)F^{p}(x)dx\right \}^{\frac{1} {p} } < \infty, {}\\ 0& <& \vert \vert g\vert \vert _{q,\varPsi } = \left \{\int _{\mathbf{R}_{+}^{j_{0}}}\varPsi (y)g^{q}(y)dy\right \}^{\frac{1} {q} } < \infty. {}\\ \end{array}$$
  1. (i)

    If p > 1, then we have the following equivalent inequalities with the best possible constant factor K(σ):

    $$\displaystyle\begin{array}{rcl} \int _{\mathbf{R}_{+}^{j_{0}}}\int _{\mathbf{R}_{+}^{i_{0}}}k_{\lambda }(\vert \vert x\vert \vert _{\alpha },\vert \vert y\vert \vert _{\beta })F(x)g(y)dxdy < K_{\lambda }(\sigma )\vert \vert F\vert \vert _{p,\varPhi }\vert \vert g\vert \vert _{q,\varPsi },& & {}\end{array}$$
    (40)
    $$\displaystyle\begin{array}{rcl} \left \{\int _{\mathbf{R}_{+}^{j_{0}}}\vert \vert y\vert \vert _{\beta }^{p\sigma -j_{0} }\left (\int _{\mathbf{R}_{+}^{i_{0}}}k_{\lambda }(\vert \vert x\vert \vert _{\alpha },\vert \vert y\vert \vert _{\beta })F(x)dx\right )^{p}dy\right \}^{\frac{1} {p} } < K_{\lambda }(\sigma )\vert \vert F\vert \vert _{p,\varPhi };& & {}\end{array}$$
    (41)
  2. (ii)

    if 0 < p < 1, or p < 0, there exists a constant δ 0 > 0, such that for any \(\tilde{\sigma }\in (\sigma -\delta _{0},\sigma +\delta _{0})\),\(k_{\lambda }(\tilde{\sigma }) \in \mathbf{R}\) , then we still have the equivalent reverses of (40) and (41) with the same best constant factor K λ (σ).

In particular, for \(i_{0} = j_{0} =\alpha =\beta = 1,\varphi _{1}(x):= x^{p(1-\mu )-1}\), if F(x) ≥ 0, g(y) ≥ 0,

$$\displaystyle\begin{array}{rcl} 0& <& \vert \vert F\vert \vert _{p,\varphi _{1}} = \left \{\int _{0}^{\infty }\varphi _{ 1}(x)F^{p}(x)dx\right \}^{\frac{1} {p} } < \infty, {}\\ 0& <& \vert \vert g\vert \vert _{q,\psi } = \left \{\int _{0}^{\infty }\psi (y)g^{q}(y)dy\right \}^{\frac{1} {q} } < \infty, {}\\ \end{array}$$

then (i) for p > 1, we have the following equivalent inequalities with the best possible constant factor k λ (σ): 

$$\displaystyle\begin{array}{rcl} \int _{0}^{\infty }\int _{ 0}^{\infty }k_{\lambda }(x,y)F(x)g(y)dxdy < k_{\lambda }(\sigma )\vert \vert F\vert \vert _{ p,\varphi _{1}}\vert \vert g\vert \vert _{q,\psi },& &{}\end{array}$$
(42)
$$\displaystyle\begin{array}{rcl} \left \{\int _{0}^{\infty }y^{p\sigma -1}\left [\int _{ 0}^{\infty }k_{\lambda }(x,y)F(x)dx\right ]^{p}dy\right \}^{\frac{1} {p} } < k_{\lambda }(\sigma )\vert \vert F\vert \vert _{p,\varphi _{ 1}};& &{}\end{array}$$
(43)
  1. (ii)

    for 0 < p < 1, or p < 0, there exists a constant δ 0 > 0, such that for any \(\tilde{\sigma }\in (\sigma -\delta _{0},\sigma +\delta _{0})\), \(k_{\lambda }(\tilde{\sigma }) \in \mathbf{R}\), we have the equivalent reverses of (42) and (43) with the same best constant factor k λ (σ).

Proof.

For δ = −1 in Theorem 1, setting h(u) = k λ (1, u) and \(\vert \vert x\vert \vert _{\alpha }^{\lambda }f(x) = F(x)\), since μ = λσ, by simplifications, we can obtain (40) and (41) (for p > 1). It is evident that (40) and (41) are equivalent with the same best constant factor K λ (σ). By the same way, we can show the cases in 0 < p < 1 or p < 0. The corollary is proved.

Remark 1.

Inequality (42), (43) is equivalent to (32), (33). In fact, Setting \(x = \frac{1} {X},h(u) = k_{\lambda }(1,u)\) in (32), (33), replacing \(X^{\lambda }f( \frac{1} {X})\) by F(X), by simplification, we obtain (42), (43). On the other hand, by (42), (43), we can deduce (32), (33).

5 Two Classes of Hardy-Type Inequalities

If h(v) = 0(v > 1), then

$$\displaystyle{h(\vert \vert x\vert \vert _{\alpha }^{\delta }\vert \vert y\vert \vert _{\beta }) = 0(\vert \vert x\vert \vert _{\alpha }^{\delta } > \vert \vert y\vert \vert _{\beta }^{-1}),}$$

by Theorem 1, we have the following first class of Hardy-type inequalities:

Corollary 4.

Suppose that \(\alpha,\beta > 0,\ \sigma \in \mathbf{R},h(v) \geq 0\),

$$\displaystyle{k_{1}(\sigma ):=\int _{ 0}^{1}h(v)v^{\sigma -1}dv \in \mathbf{R}_{ +},}$$
$$\displaystyle{H_{1}(\sigma ):= \left [ \frac{\varGamma ^{j_{0}}\big(\frac{1} {\beta } \big)} {\beta ^{j_{0}-1}\varGamma \big(\frac{j_{0}} {\beta } \big)}\right ]^{\frac{1} {p} }\left [ \frac{\varGamma ^{i_{0}}\big(\frac{1} {\alpha } \big)} {\alpha ^{i_{0}-1}\varGamma \big(\frac{i_{0}} {\alpha } \big)}\right ]^{\frac{1} {q} }k_{1}(\sigma ),}$$

\(\delta \in \{-1,1\},p \in \mathbf{R}\setminus \{0,1\}\),\(\frac{1} {p} + \frac{1} {q} = 1\),\(f(x) = f(x_{1},\ldots,x_{i_{0}}) \geq 0\),\(g(y) = g(y_{1},\ldots,y_{j_{0}}) \geq 0\),

$$\displaystyle\begin{array}{rcl} 0& <& \vert \vert f\vert \vert _{p,\varPhi _{\delta }} = \left \{\int _{\mathbf{R}_{+}^{i_{0}}}\varPhi _{\delta }(x)f^{p}(x)dx\right \}^{\frac{1} {p} } < \infty, {}\\ 0& <& \vert \vert g\vert \vert _{q,\varPsi } = \left \{\int _{\mathbf{R}_{+}^{j_{0}}}\varPsi (y)g^{q}(y)dy\right \}^{\frac{1} {q} } < \infty. {}\\ \end{array}$$
  1. (i)

    If p > 1, then we have the following equivalent inequalities with the best possible constant factor H 1 (σ):

    $$\displaystyle\begin{array}{rcl} & \int _{\mathbf{R}_{+}^{j_{0}}}\left [\int _{\left \{x\in \mathbf{R}_{+}^{i_{0}};\vert \vert x\vert \vert _{\alpha }^{\delta }\leq \vert \vert y\vert \vert _{\beta }^{-1}\right \}}h(\vert \vert x\vert \vert _{\alpha }^{\delta }\vert \vert y\vert \vert _{\beta })f(x)dx\right ]g(y)dy& \\ & < H_{1}(\sigma )\vert \vert f\vert \vert _{p,\varPhi _{\delta }}\vert \vert g\vert \vert _{q,\varPsi }, & {}\end{array}$$
    (44)
    $$\displaystyle\begin{array}{rcl} & \left \{\int _{\mathbf{R}_{+}^{j_{0}}}\vert \vert y\vert \vert _{\beta }^{p\sigma -j_{0}}\left (\int _{\left \{x\in \mathbf{R}_{ +}^{i_{0}};\vert \vert x\vert \vert _{\alpha }^{\delta }\leq \vert \vert y\vert \vert _{\beta }^{-1}\right \}}h\left (\vert \vert x\vert \vert _{\alpha }^{\delta }\vert \vert y\vert \vert _{\beta }\right )f(x)dx\right )^{p}dy\right \}^{\frac{1} {p} }& \\ & < H_{1}(\sigma )\vert \vert f\vert \vert _{p,\varPhi _{\delta }}; & {}\end{array}$$
    (45)
  2. (ii)

    If 0 < p < 1, or p < 0, there exists a constant δ 0 > 0, such that for any \(\tilde{\sigma }\in (\sigma -\delta _{0},\sigma +\delta _{0})\),\(k_{1}(\tilde{\sigma }) \in \mathbf{R}\) , then we still have the equivalent reverses of (44) and (45) with the same best constant factor H 1 (σ).

For i 0 = j 0 = α = β = 1, δ = 1 in Corollary 4, we have

Corollary 5.

Assuming that \(\sigma \in \mathbf{R},k_{1}(\sigma ) \in \mathbf{R}_{+},p \in \mathbf{R}\setminus \{0,1\}, \frac{1} {p} + \frac{1} {q} = 1\) , we set

$$\displaystyle{\varphi (x):= x^{p(1-\sigma )-1},\psi (y):= y^{q(1-\sigma )-1}(x,y > 0).}$$

If f(x) ≥ 0, g(y) ≥ 0,

$$\displaystyle\begin{array}{rcl} 0& <& \vert \vert f\vert \vert _{p,\varphi } = \left \{\int _{0}^{\infty }\varphi (x)f^{p}(x)dx\right \}^{\frac{1} {p} } < \infty, {}\\ 0& <& \vert \vert g\vert \vert _{q,\psi } = \left \{\int _{0}^{\infty }\psi (y)g^{q}(y)dy\right \}^{\frac{1} {q} } < \infty, {}\\ \end{array}$$

then (i) for p > 1, we have the following equivalent inequalities with the best possible constant factor k 1 (σ):

$$\displaystyle\begin{array}{rcl} \int _{0}^{\infty }\left (\int _{ 0}^{ \frac{1} {y} }h(xy)f(x)dx\right )g(y)dy < k_{1}(\sigma )\vert \vert f\vert \vert _{p,\varphi }\vert \vert g\vert \vert _{q,\psi },& &{}\end{array}$$
(46)
$$\displaystyle\begin{array}{rcl} \left \{\int _{0}^{\infty }y^{p\sigma -1}\left [\int _{ 0}^{ \frac{1} {y} }h(xy)f(x)dx\right ]^{p}dy\right \}^{\frac{1} {p} } < k_{1}(\sigma )\vert \vert f\vert \vert _{p,\varphi };& & {}\end{array}$$
(47)

(ii) for 0 < p < 1, or p < 0, there exists a constant δ 0 > 0, such that for any \(\tilde{\sigma }\in (\sigma -\delta _{0},\sigma +\delta _{0})\),\(k_{1}(\tilde{\sigma }) \in \mathbf{R}\) , we have the equivalent reverses of (46) and (47) with the same best constant factor k 1 (σ).

If k λ (x, y) = 0(x < y), by (42) and (43), we have

Corollary 6.

Assuming that μ,σ ∈ R ,μ + σ = λ,

$$\displaystyle{k_{\lambda }^{(1)}(\sigma ):=\int _{ 0}^{1}k_{\lambda }(1,v)v^{\sigma -1}dv \in \mathbf{R}_{ +},}$$

\(p \in \mathbf{R}\setminus \{0,1\}, \frac{1} {p} + \frac{1} {q} = 1\),\(\varphi _{1}(x):= x^{p(1-\mu )-1}\) , if F(x) ≥ 0, g(y) ≥ 0,

$$\displaystyle\begin{array}{rcl} 0& <& \vert \vert F\vert \vert _{p,\varphi _{1}} = \left \{\int _{0}^{\infty }\varphi _{ 1}(x)F^{p}(x)dx\right \}^{\frac{1} {p} } < \infty, {}\\ 0& <& \vert \vert g\vert \vert _{q,\psi } = \left \{\int _{0}^{\infty }\psi (y)g^{q}(y)dy\right \}^{\frac{1} {q} } < \infty, {}\\ \end{array}$$

then (i) for p > 1, we have the following equivalent inequalities with the best possible constant factor \(k_{\lambda }^{(1)}(\sigma ):\)

$$\displaystyle\begin{array}{rcl} \int _{0}^{\infty }\left [\int _{ y}^{\infty }k_{\lambda }(x,y)F(x)dx\right ]g(y)dy < k_{\lambda }^{(1)}(\sigma )\vert \vert F\vert \vert _{ p,\varphi _{1}}\vert \vert g\vert \vert _{q,\psi },& &{}\end{array}$$
(48)
$$\displaystyle\begin{array}{rcl} \left \{\int _{0}^{\infty }y^{p\sigma -1}\left [\int _{ y}^{\infty }k_{\lambda }(x,y)F(x)dx\right ]^{p}dy\right \}^{\frac{1} {p} } < k_{\lambda }^{(1)}(\sigma )\vert \vert F\vert \vert _{p,\varphi _{ 1}};& & {}\end{array}$$
(49)

(ii) for 0 < p < 1, or p < 0, there exists a constant δ 0 > 0, such that for any \(\tilde{\sigma }\in (\sigma -\delta _{0},\sigma +\delta _{0})\),\(k_{\lambda }^{(1)}(\tilde{\sigma }) \in \mathbf{R}\) , we have the equivalent reverses of (48) and (49) with the same best constant factor \(k_{\lambda }^{(1)}(\sigma )\) .

If h(v) = 0(0 < v < 1), then

$$\displaystyle{h(\vert \vert x\vert \vert _{\alpha }^{\delta }\vert \vert y\vert \vert _{\beta }) = 0(\vert \vert x\vert \vert _{\alpha }^{\delta } < \vert \vert y\vert \vert _{\beta }^{-1}),}$$

by Theorem 1, we have the following second class of Hardy-type inequalities:

Corollary 7.

Suppose that \(\alpha,\beta > 0,\ \sigma \in \mathbf{R},h(v) \geq 0\),

$$\displaystyle\begin{array}{rcl} & k_{2}(\sigma ):=\int _{ 1}^{\infty }h(v)v^{\sigma -1}dv \in \mathbf{R}_{+}, & {}\\ & H_{2}(\sigma ):= \left [ \frac{\varGamma ^{j_{0}}\big(\frac{1} {\beta } \big)} {\beta ^{j_{0}-1}\varGamma \big(\frac{j_{0}} {\beta } \big)}\right ]^{\frac{1} {p} }\left [ \frac{\varGamma ^{i_{0}}\big(\frac{1} {\alpha } \big)} {\alpha ^{i_{0}-1}\varGamma \big(\frac{i_{0}} {\alpha } \big)}\right ]^{\frac{1} {q} }k_{2}(\sigma ),& {}\\ \end{array}$$

\(\delta \in \{-1,1\},p \in \mathbf{R}\setminus \{0,1\}\),\(\frac{1} {p} + \frac{1} {q} = 1\),\(f(x) = f(x_{1},\ldots,x_{i_{0}}) \geq 0\),\(g(y) = g(y_{1},\ldots,y_{j_{0}}) \geq 0\),

$$\displaystyle\begin{array}{rcl} 0& <& \vert \vert f\vert \vert _{p,\varPhi _{\delta }} = \left \{\int _{\mathbf{R}_{+}^{i_{0}}}\varPhi _{\delta }(x)f^{p}(x)dx\right \}^{\frac{1} {p} } < \infty, {}\\ 0& <& \vert \vert g\vert \vert _{q,\varPsi } = \left \{\int _{\mathbf{R}_{+}^{j_{0}}}\varPsi (y)g^{q}(y)dy\right \}^{\frac{1} {q} } < \infty. {}\\ \end{array}$$
  1. (i)

    If p > 1, then we have the following equivalent inequalities with the best possible constant factor H 2 (σ):

    $$\displaystyle\begin{array}{rcl} & \int _{\mathbf{R}_{+}^{j_{0}}}\left [\int _{\left \{x\in \mathbf{R}_{+}^{i_{0}};\vert \vert x\vert \vert _{\alpha }^{\delta }\geq \vert \vert y\vert \vert _{\beta }^{-1}\right \}}h\left (\vert \vert x\vert \vert _{\alpha }^{\delta }\vert \vert y\vert \vert _{\beta }\right )f(x)dx\right ]g(y)dy& \\ & < H_{2}(\sigma )\vert \vert f\vert \vert _{p,\varPhi _{\delta }}\vert \vert g\vert \vert _{q,\varPsi }, & {}\end{array}$$
    (50)
    $$\displaystyle\begin{array}{rcl} & \left \{\int _{\mathbf{R}_{+}^{j_{0}}}\vert \vert y\vert \vert _{\beta }^{p\sigma -j_{0}}\left (\int _{\left \{x\in \mathbf{R}_{ +}^{i_{0}};\vert \vert x\vert \vert _{\alpha }^{\delta }\geq \vert \vert y\vert \vert _{\beta }^{-1}\right \}}h(\vert \vert x\vert \vert _{\alpha }^{\delta }\vert \vert y\vert \vert _{\beta })f(x)dx\right )^{p}dy\right \}^{\frac{1} {p} }& \\ & < H_{2}(\sigma )\vert \vert f\vert \vert _{p,\varPhi _{\delta }}; & {}\end{array}$$
    (51)
  2. (ii)

    if 0 < p < 1, or p < 0, there exists a constant δ 0 > 0, such that for any \(\tilde{\sigma }\in (\sigma -\delta _{0},\sigma +\delta _{0})\),\(k_{2}(\tilde{\sigma }) \in \mathbf{R}\) , then we still have the equivalent reverses of (50) and (51) with the same best constant factor H 2 (σ).

For \(i_{0} = j_{0} =\alpha =\beta = 1,\delta = 1\) in Corollary 7, we have

Corollary 8.

Assuming that \(\sigma \in \mathbf{R},k_{2}(\sigma ) \in \mathbf{R}_{+},p \in \mathbf{R}\setminus \{0,1\}, \frac{1} {p} + \frac{1} {q} = 1\) , we set

$$\displaystyle{\varphi (x) = x^{p(1-\sigma )-1},\psi (y) = y^{q(1-\sigma )-1}(x,y > 0).}$$

If f(x) ≥ 0, g(y) ≥ 0,

$$\displaystyle\begin{array}{rcl} 0& <& \vert \vert f\vert \vert _{p,\varphi } = \left \{\int _{0}^{\infty }\varphi (x)f^{p}(x)dx\right \}^{\frac{1} {p} } < \infty, {}\\ 0& <& \vert \vert g\vert \vert _{q,\psi } = \left \{\int _{0}^{\infty }\psi (y)g^{q}(y)dy\right \}^{\frac{1} {q} } < \infty. {}\\ \end{array}$$

then (i) for p > 1, we have the following equivalent inequalities with the best possible constant factor k 2 (σ):

$$\displaystyle\begin{array}{rcl} \int _{0}^{\infty }\left (\int _{\frac{ 1} {y} }^{\infty }h(xy)f(x)dx\right )g(y)dy < k_{ 2}(\sigma )\vert \vert f\vert \vert _{p,\varphi }\vert \vert g\vert \vert _{q,\psi },& &{}\end{array}$$
(52)
$$\displaystyle\begin{array}{rcl} \left \{\int _{0}^{\infty }y^{p\sigma -1}\left [\int _{\frac{ 1} {y} }^{\infty }h(xy)f(x)dx\right ]^{p}dy\right \}^{\frac{1} {p} } < k_{2}(\sigma )\vert \vert f\vert \vert _{p,\varphi };& & {}\end{array}$$
(53)

(ii) for 0 < p < 1, or p < 0, there exists a constant δ 0 > 0, such that for any \(\tilde{\sigma }\in (\sigma -\delta _{0},\sigma +\delta _{0})\),\(k_{2}(\tilde{\sigma }) \in \mathbf{R}\) , we have the equivalent reverses of (52) and (53) with the same best constant factor k 2 (σ).

If k λ (x, y) = 0(x > y), by (42) and (43), we have

Corollary 9.

Assuming that μ,σ ∈ R ,μ + σ = λ,

$$\displaystyle{k_{\lambda }^{(2)}(\sigma ):=\int _{ 1}^{\infty }k_{\lambda }(1,v)v^{\sigma -1}dv \in \mathbf{R}_{ +},}$$

\(p \in \mathbf{R}\setminus \{0,1\}, \frac{1} {p} + \frac{1} {q} = 1\),\(\varphi _{1}(x):= x^{p(1-\mu )-1}\) , if F(x) ≥ 0, g(y) ≥ 0,

$$\displaystyle\begin{array}{rcl} 0& <& \vert \vert F\vert \vert _{p,\varphi _{1}} = \left \{\int _{0}^{\infty }\varphi _{ 1}(x)F^{p}(x)dx\right \}^{\frac{1} {p} } < \infty, {}\\ 0& <& \vert \vert g\vert \vert _{q,\psi } = \left \{\int _{0}^{\infty }\psi (y)g^{q}(y)dy\right \}^{\frac{1} {q} } < \infty, {}\\ \end{array}$$

then (i) for p > 1, we have the following equivalent inequalities with the best possible constant factor \(k_{\lambda }^{(2)}(\sigma ):\)

$$\displaystyle\begin{array}{rcl} \int _{0}^{\infty }\left [\int _{ 0}^{y}k_{\lambda }(x,y)F(x)dx\right ]g(y)dy < k_{\lambda }^{(2)}(\sigma )\vert \vert F\vert \vert _{ p,\varphi _{1}}\vert \vert g\vert \vert _{q,\psi },& &{}\end{array}$$
(54)
$$\displaystyle\begin{array}{rcl} \left \{\int _{0}^{\infty }y^{p\sigma -1}\left [\int _{ 0}^{y}k_{\lambda }(x,y)F(x)dx\right ]^{p}dy\right \}^{\frac{1} {p} } < k_{\lambda }^{(2)}(\sigma )\vert \vert F\vert \vert _{p,\varphi _{ 1}};& & {}\end{array}$$
(55)

(ii) for 0 < p < 1, or p < 0, there exists a constant δ 0 > 0, such that for any \(\tilde{\sigma }\in (\sigma -\delta _{0},\sigma +\delta _{0})\),\(k_{\lambda }^{(2)}(\tilde{\sigma }) \in \mathbf{R}\) , we have the equivalent reverses of (54) and (55) with the same best constant factor \(k_{\lambda }^{(2)}(\sigma )\) .

6 Multidimensional Hilbert-Type Inequalities with Two Variables

Suppose that \(u_{i}(s_{i}),u_{i}^{{\prime}}(s_{i}) > 0,u_{i}(a_{i}^{+}) = 0,u_{i}(b_{i}^{-}) = \infty (-\infty \leq a_{i} < b_{i} \leq \infty,i = 1,\ldots,i_{0})\), \(u(s) = (u_{1}(s_{1}),\ldots,u_{i_{0}}(s_{i_{0}})),v_{j}(t_{j}),v_{j}^{{\prime}}(t_{j}) > 0,v_{j}(c_{j}^{+}) = 0,v_{j}(d_{j}^{-}) = \infty \,\,\) \(\,\,(-\infty \leq c_{j} < d_{j} \leq \infty,j = 1,\ldots,j_{0})\),

\(v(t) = (v_{1}(t_{1}),\ldots,v_{j_{0}}(t_{j_{0}}))\),

$$\displaystyle{\tilde{\varPhi }_{\delta }(s):= \frac{\vert \vert u(s)\vert \vert _{\alpha }^{p(i_{0}-\delta \sigma )-i_{0}}} {\left [\varPi _{i=1}^{i_{0}}u_{i}^{{\prime}}(s_{i})\right ]^{p-1}},\tilde{\varPsi }(t):= \frac{\vert \vert v(t)\vert \vert _{\alpha }^{q(j_{0}-\sigma )-j_{0}}} {\left [\varPi _{j=1}^{j_{0}}v_{j}^{{\prime}}(t_{j})\right ]^{q-1}}.}$$

Setting x = u(s), y = v(t) in Theorem 1, for

$$\displaystyle{F(s):=\varPi _{ i=1}^{i_{0} }u_{i}^{{\prime}}(s_{ i})f(u(s)),G(t):=\varPi _{ j=1}^{j_{0} }v_{j}^{{\prime}}(t_{ j})g(v(t)),}$$

we have

Theorem 2.

Suppose that \(\alpha,\beta > 0,\ \sigma \in \mathbf{R},h(v) \geq 0\),

$$\displaystyle\begin{array}{rcl} & k(\sigma ) =\int _{ 0}^{\infty }h(v)v^{\sigma -1}dv \in \mathbf{R}_{+}, & {}\\ & K(\sigma ) = \left [ \frac{\varGamma ^{j_{0}}\big(\frac{1} {\beta } \big)} {\beta ^{j_{0}-1}\varGamma \big(\frac{j_{0}} {\beta } \big)}\right ]^{\frac{1} {p} }\left [ \frac{\varGamma ^{i_{0}}\big(\frac{1} {\alpha } \big)} {\alpha ^{i_{0}-1}\varGamma \big(\frac{i_{0}} {\alpha } \big)}\right ]^{\frac{1} {q} }k(\sigma ),& {}\\ \end{array}$$

\(\delta \in \{-1,1\},p \in \mathbf{R}\setminus \{0,1\}\),\(\frac{1} {p} + \frac{1} {q} = 1\),\(F(s) = F(s_{1},\ldots,s_{i_{0}}) \geq 0\),\(G(t) = G(t_{1},\ldots,t_{j_{0}})\) ≥ 0,

$$\displaystyle\begin{array}{rcl} 0& <& \vert \vert F\vert \vert _{p,\tilde{\varPhi }_{\delta }} = \left \{\int _{\left \{s\in \mathbf{R}^{i_{0}};a_{i}<s_{i}<b_{i}\right \}}\tilde{\varPhi }_{\delta }(s)F^{p}(s)ds\right \}^{\frac{1} {p} } < \infty, {}\\ 0& <& \vert \vert G\vert \vert _{q,\tilde{\varPsi }} = \left \{\int _{\left \{t\in \mathbf{R}^{j_{0}};c_{j}<t_{j}<d_{j}\right \}}\tilde{\varPsi }(t)G^{q}(t)dt\right \}^{\frac{1} {q} } < \infty. {}\\ \end{array}$$
  1. (i)

    If p > 1, then we have the following equivalent inequalities with the best possible constant factor K(σ):

    $$\displaystyle\begin{array}{rcl} & \int _{\left \{t\in \mathbf{R}^{j_{0}};c_{j}<t_{j}<d_{j}\right \}}\int _{\left \{s\in \mathbf{R}^{i_{0}};a_{i}<s_{i}<b_{i}\right \}}h(\vert \vert u(s)\vert \vert _{\alpha }^{\delta }\vert \vert v(t)\vert \vert _{\beta })F(s)G(t)dsdt& \\ & < K(\sigma )\vert \vert F\vert \vert _{p,\tilde{\varPhi }_{\delta }}\vert \vert g\vert \vert _{q,\tilde{\varPsi }}, & {}\end{array}$$
    (56)
    $$\displaystyle\begin{array}{rcl} & \left \{\int _{\{t\in \mathbf{R}^{j_{0}};c_{j}<t_{j}<d_{j}\}}\vert \vert v(t)\vert \vert _{\beta }^{p\sigma -j_{0}}\varPi _{j=1}^{j_{0}}v_{j}^{{\prime}}(t_{j})\left (\int _{\{s\in \mathbf{R}^{i_{ 0}};a_{i}<s_{i}<b_{i}\}}h(\vert \vert u(s)\vert \vert _{\alpha }^{\delta }\vert \vert v(t)\vert \vert _{\beta })\right.\right.& \\ & \left.\left.F(s)ds\right )^{p}dt\right \}^{\frac{1} {p} } < K(\sigma )\vert \vert F\vert \vert _{p,\tilde{\varPhi }_{\delta }}; & {}\end{array}$$
    (57)
  2. (ii)

    if 0 < p < 1, or p < 0, there exists a constant δ 0 > 0, such that for any \(\tilde{\sigma }\in (\sigma -\delta _{0},\sigma +\delta _{0})\),\(k(\tilde{\sigma }) \in \mathbf{R}\) , then we still have the equivalent reverses of (56) and (57) with the same best constant factor K(σ).

In particular, for i 0 = j 0 = α = β = 1,

$$\displaystyle\begin{array}{rcl} & \tilde{\phi }_{\delta }(s):= \frac{(u(s))^{p(1-\delta \sigma )-1}} {[u^{{\prime}}(s)]^{p-1}},\tilde{\psi }(t):= \frac{(v(t))^{q(1-\sigma )-1}} {[v^{{\prime}}(t)]^{q-1}}, & {}\\ & \begin{array}{ll} 0& < \vert \vert F\vert \vert _{p,\tilde{\phi }_{\delta }} = \left \{\int _{a}^{b}\tilde{\phi }_{\delta }(s)F^{p}(s)ds\right \}^{\frac{1} {p} } < \infty, \\ 0& < \vert \vert G\vert \vert _{q,\tilde{\psi }} = \left \{\int _{c}^{d}\tilde{\varPsi }(t)G^{q}(t)dt\right \}^{\frac{1} {q} } < \infty,\end{array} & {}\\ \end{array}$$
  1. (i)

    if p > 1, then we have the following equivalent inequalities with the best possible constant factor k(σ): 

    $$\displaystyle\begin{array}{rcl} \int _{c}^{d}\int _{ a}^{b}h(u^{\delta }(s)v(t))F(s)G(t)dsdt < k(\sigma )\vert \vert F\vert \vert _{ p,\tilde{\phi }_{\delta }}\vert \vert G\vert \vert _{q,\tilde{\psi }},& & {}\end{array}$$
    (58)
    $$\displaystyle\begin{array}{rcl} \left \{\int _{c}^{d}(v(t))^{p\sigma -1}v^{{\prime}}(t)\left (\int _{ a}^{b}h(u^{\delta }(s)v(t))F(s)ds\right )^{p}dt\right \}^{\frac{1} {p} } < k(\sigma )\vert \vert F\vert \vert _{p,\tilde{\phi }_{\delta }};& & {}\end{array}$$
    (59)
  2. (ii)

    if 0 < p < 1, or p < 0, there exists a constant δ 0 > 0, such that for any \(\tilde{\sigma }\in (\sigma -\delta _{0},\sigma +\delta _{0})\), \(k(\tilde{\sigma }) \in \mathbf{R}\), then we still have the equivalent reverses of (58) and (59) with the same best constant factor k(σ).

In particular, for \(\gamma,\eta > 0,u_{i}(s_{i}) = s_{i}^{\gamma },u_{i}^{{\prime}}(s_{i}) =\gamma s_{i}^{\gamma -1},u_{i}(0^{+}) = 0,u_{i}(\infty ) = \infty (a_{i} = 0,b_{i} = \infty,i = 1,\ldots,i_{0})\), \(\hat{u}(s) = (s_{1}^{\gamma },\ldots,s_{i_{0}}^{\gamma }),v_{ j}(t_{j}) = t_{j}^{\eta },v_{ j}^{{\prime}}(t_{ j}) =\eta t_{j}^{\eta -1},v_{ j}(0^{+}) = 0,v_{ j}(\infty ) = \infty (c_{j} = 0,d_{j} = \infty,j = 1,\ldots,j_{0})\), \(\hat{v}(t) = (t_{1}^{\eta },\ldots,t_{j_{0}}^{\eta })\), and

$$\displaystyle\begin{array}{rcl} \tilde{\varPhi }_{\delta }(s)& =& \frac{1} {\gamma ^{i_{0}(p-1)}}\hat{\varPhi }_{\delta }(s),\hat{\varPhi }_{\delta }(s):= \frac{\vert \vert \hat{u}(s)\vert \vert _{\alpha }^{p(i_{0}-\delta \sigma )-i_{0}}} {\left (\varPi _{i=1}^{i_{0}}s_{i}^{\gamma -1}\right )^{p-1}}, {}\\ \tilde{\varPsi }(t)& =& \frac{1} {\eta ^{j_{0}(q-1)}}\hat{\varPsi }(t),\hat{\varPsi }(t):= \frac{\vert \vert \hat{v}(t)\vert \vert _{\alpha }^{q(j_{0}-\sigma )-j_{0}}} {\left (\varPi _{j=1}^{j_{0}}t_{j}^{\eta -1}\right )^{q-1}} {}\\ \end{array}$$

in Theorem 2, we have

Corollary 10.

Suppose that \(\alpha,\beta,\gamma,\eta > 0,\ \sigma \in \mathbf{R},h(v) \geq 0\),

$$\displaystyle\begin{array}{rcl} & k(\sigma ) =\int _{ 0}^{\infty }h(v)v^{\sigma -1}dv \in \mathbf{R}_{+}, & {}\\ & K(\sigma ) = \left [ \frac{\varGamma ^{j_{0}}\big(\frac{1} {\beta } \big)} {\beta ^{j_{0}-1}\varGamma \big(\frac{j_{0}} {\beta } \big)}\right ]^{\frac{1} {p} }\left [ \frac{\varGamma ^{i_{0}}\big(\frac{1} {\alpha } \big)} {\alpha ^{i_{0}-1}\varGamma \big(\frac{i_{0}} {\alpha } \big)}\right ]^{\frac{1} {q} }k(\sigma ),& {}\\ \end{array}$$

\(\delta \in \{-1,1\},p \in \mathbf{R}\setminus \{0,1\}\),\(\frac{1} {p} + \frac{1} {q} = 1\),\(F(s) = F(s_{1},\ldots,s_{i_{0}}) \geq 0\),\(G(t) = G(t_{1},\ldots,t_{j_{0}})\) ≥ 0,

$$\displaystyle\begin{array}{rcl} 0& <& \vert \vert F\vert \vert _{p,\hat{\varPhi }_{\delta }} = \left \{\int _{\mathbf{R}_{+}^{i_{0}}}\hat{\varPhi }_{\delta }(s)F^{p}(s)ds\right \}^{\frac{1} {p} } < \infty, {}\\ 0& <& \vert \vert G\vert \vert _{q,\hat{\varPsi }} = \left \{\int _{\mathbf{R}_{+}^{j_{0}}}\hat{\varPsi }(t)G^{q}(t)dt\right \}^{\frac{1} {q} } < \infty. {}\\ \end{array}$$
  1. (i)

    If p > 1, then we have the following equivalent inequalities with the best possible constant factor \(\frac{1} {\gamma ^{i_{0}/q}\eta ^{j_{0}/p}}K(\sigma ):\)

    $$\displaystyle\begin{array}{rcl} & \int _{\mathbf{R}_{+}^{j_{0}}}\int _{\mathbf{R}_{+}^{i_{0}}}h(\vert \vert \hat{u}(s)\vert \vert _{\alpha }^{\delta }\vert \vert \hat{v}(t)\vert \vert _{\beta })F(s)G(t)dsdt& \\ & < \frac{1} {\gamma ^{i_{0}/q}\eta ^{j_{0}/p}}K(\sigma )\vert \vert F\vert \vert _{p,\hat{\varPhi }_{\delta }}\vert \vert G\vert \vert _{q,\hat{\varPsi }}, & {}\end{array}$$
    (60)
    $$\displaystyle\begin{array}{rcl} & \left \{\int _{\mathbf{R}_{+}^{j_{0}}}\vert \vert \hat{v}(t)\vert \vert _{\beta }^{p\sigma -j_{0}}\varPi _{j=1}^{j_{0}}t_{j}^{\eta -1}\left (\int _{\mathbf{R}_{ +}^{i_{0}}}h(\vert \vert \hat{u}(s)\vert \vert _{\alpha }^{\delta }\vert \vert \hat{v}(t)\vert \vert _{\beta })\right.\right.& \\ & \left.\left.F(s)ds\right )^{p}dt\right \}^{\frac{1} {p} } < \frac{1} {\gamma ^{i_{0}/q}\eta ^{j_{0}/p}}K(\sigma )\vert \vert F\vert \vert _{p,\hat{\varPhi }_{\delta }}; & {}\end{array}$$
    (61)
  2. (ii)

    if 0 < p < 1, or p < 0, there exists a constant δ 0 > 0, such that for any \(\tilde{\sigma }\in (\sigma -\delta _{0},\sigma +\delta _{0})\),\(k(\tilde{\sigma }) \in \mathbf{R}\) , then we still have the equivalent reverses of (60) and (61) with the same best constant factor \(\frac{1} {\gamma ^{i_{0}/q}\eta ^{j_{0}/p}}K(\sigma )\) .

In particular, for i 0 = j 0 = α = β = 1,

$$\displaystyle\begin{array}{rcl} & \hat{\phi }_{\delta }(s):= s^{p(1-\delta \gamma \sigma )-1},\hat{\psi }(t):= t^{q(1-\eta \sigma )-1}, & {}\\ & \begin{array}{ll} 0& < \vert \vert F\vert \vert _{p,\hat{\phi }_{\delta }} = \left \{\int _{0}^{\infty }\hat{\phi }_{\delta }(s)F^{p}(s)ds\right \}^{\frac{1} {p} } < \infty, \\ 0& < \vert \vert G\vert \vert _{q,\hat{\psi }} = \left \{\int _{0}^{\infty }\hat{\psi }(t)G^{q}(t)dt\right \}^{\frac{1} {q} } < \infty,\end{array} & {}\\ \end{array}$$
  1. (i)

    if p > 1, then we have the following equivalent inequalities with the best possible constant factor \(\frac{1} {\gamma ^{1/q}\eta ^{1/p}}k(\sigma ):\)

    $$\displaystyle\begin{array}{rcl} \int _{0}^{\infty }\int _{ 0}^{\infty }h(s^{\gamma \delta }t^{\eta })F(s)G(t)dsdt < \frac{1} {\gamma ^{1/q}\eta ^{1/p}}k(\sigma )\vert \vert F\vert \vert _{p,\hat{\phi }_{\delta }}\vert \vert G\vert \vert _{q,\hat{\psi }},& & {}\end{array}$$
    (62)
    $$\displaystyle\begin{array}{rcl} \left \{\int _{0}^{\infty }t^{p\eta \sigma -1}\left (\int _{ 0}^{\infty }h(s^{\gamma \delta }t^{\eta })F(s)ds\right )^{p}dt\right \}^{\frac{1} {p} } < \frac{1} {\gamma ^{1/q}\eta ^{1/p}}k(\sigma )\vert \vert F\vert \vert _{p,\hat{\phi }_{\delta }};& & {}\end{array}$$
    (63)
  2. (ii)

    if 0 < p < 1, or p < 0, there exists a constant δ 0 > 0, such that for any \(\tilde{\sigma }\in (\sigma -\delta _{0},\sigma +\delta _{0})\), \(k(\tilde{\sigma }) \in \mathbf{R}\), then we still have the equivalent reverses of (62) and (63) with the same best constant factor \(\frac{1} {\gamma ^{1/q}\eta ^{1/p}}k(\sigma )\).

For \(\delta = -1,h(u) = k_{\lambda }(1,u)\), \(\vert \vert u(s)\vert \vert _{\alpha }^{\lambda }F(s) = f(s)\), μ = λσ and

$$\displaystyle{\tilde{\varPhi }(s):= \frac{\vert \vert u(s)\vert \vert _{\alpha }^{p(i_{0}-\mu )-i_{0}}} {\left [\varPi _{i=1}^{i_{0}}u_{i}^{{\prime}}(s_{i})\right ]^{p-1}}}$$

in Theorem 2, by simplifications, we have

Corollary 11.

Suppose that \(\alpha,\beta > 0,\ \lambda,\mu,\sigma \in \mathbf{R},\mu +\sigma =\lambda,k_{\lambda }(x,y)(\geq 0)\) is a homogeneous function of degree −λ in \(\mathbf{R}_{+}^{2}\) , with

$$\displaystyle\begin{array}{rcl} & k_{\lambda }(\sigma ) =\int _{ 0}^{\infty }k_{\lambda }(1,v)v^{\sigma -1}dv \in \mathbf{R}_{+}, & {}\\ & K_{\lambda }(\sigma ) = \left [ \frac{\varGamma ^{j_{0}}\big(\frac{1} {\beta } \big)} {\beta ^{j_{0}-1}\varGamma \big(\frac{j_{0}} {\beta } \big)}\right ]^{\frac{1} {p} }\left [ \frac{\varGamma ^{i_{0}}\big(\frac{1} {\alpha } \big)} {\alpha ^{i_{0}-1}\varGamma \big(\frac{i_{0}} {\alpha } \big)}\right ]^{\frac{1} {q} }k_{\lambda }(\sigma ),& {}\\ \end{array}$$

p ∈ R ∖{0,1}, \(\frac{1} {p} + \frac{1} {q} = 1\),\(f(s) = f(s_{1},\ldots,s_{i_{0}}) \geq 0\),\(G(t) = G(t_{1},\ldots,t_{j_{0}}) \geq 0\),

$$\displaystyle\begin{array}{rcl} 0& <& \vert \vert f\vert \vert _{p,\tilde{\varPhi }} = \left \{\int _{\left \{s\in \mathbf{R}^{i_{0}};a_{i}<s_{i}<b_{i}\right \}}\tilde{\varPhi }(s)f^{p}(s)ds\right \}^{\frac{1} {p} } < \infty, {}\\ 0& <& \vert \vert G\vert \vert _{q,\tilde{\varPsi }} = \left \{\int _{\left \{t\in \mathbf{R}^{j_{0}};c_{j}<t_{j}<d_{j}\right \}}\tilde{\varPsi }(t)G^{q}(t)dt\right \}^{\frac{1} {q} } < \infty. {}\\ \end{array}$$
  1. (i)

    If p > 1, then we have the following equivalent inequalities with the best possible constant factor K(σ):

    $$\displaystyle\begin{array}{rcl} & \int _{\left \{t\in \mathbf{R}^{j_{0}};c_{j}<t_{j}<d_{j}\right \}}\int _{\left \{s\in \mathbf{R}^{i_{0}};a_{i}<s_{i}<b_{i}\right \}}k_{\lambda }(\vert \vert u(s)\vert \vert _{\alpha },\vert \vert v(t)\vert \vert _{\beta })f(s)G(t)dsdt& \\ & < K_{\lambda }(\sigma )\vert \vert f\vert \vert _{p,\tilde{\varPhi }}\vert \vert G\vert \vert _{q,\tilde{\varPsi }}, & {}\end{array}$$
    (64)
    $$\displaystyle\begin{array}{rcl} & \left \{\int _{\left \{t\in \mathbf{R}^{j_{0}};c_{j}<t_{j}<d_{j}\right \}}\vert \vert v(t)\vert \vert _{\beta }^{p\sigma -j_{0}}\varPi _{j=1}^{j_{0}}v_{j}^{{\prime}}(t_{j})\left (\int _{\left \{s\in \mathbf{R}^{i_{ 0}};a_{i}<s_{i}<b_{i}\right \}}k_{\lambda }(\vert \vert u(s)\vert \vert _{\alpha },\vert \vert v(t)\vert \vert _{\beta })\right.\right.& \\ & \times \left.\left.f(s)ds\right )^{p}dt\right \}^{\frac{1} {p} } < K_{\lambda }(\sigma )\vert \vert f\vert \vert _{p,\tilde{\varPhi }}; & {}\end{array}$$
    (65)
  2. (ii)

    if 0 < p < 1, or p < 0, there exists a constant δ 0 > 0, such that for any \(\tilde{\sigma }\in (\sigma -\delta _{0},\sigma +\delta _{0})\),\(k_{\lambda }(\tilde{\sigma }) \in \mathbf{R}\) , then we still have the equivalent reverses of (64) and (65) with the same best constant factor K λ (σ).

In particular, for i 0 = j 0 = α = β = 1,

$$\displaystyle\begin{array}{rcl} & \tilde{\phi }(s):= \frac{(u(s))^{p(1-\mu )-1}} {\left [u^{{\prime}}(s)\right ]^{p-1}},\tilde{\psi }(t) = \frac{(v(t))^{q(1-\sigma )-1}} {\left [v^{{\prime}}(t)\right ]^{q-1}}, & {}\\ & \begin{array}{ll} 0& < \vert \vert f\vert \vert _{p,\tilde{\phi }} = \left \{\int _{a}^{b}\tilde{\phi }(s)f^{p}(s)ds\right \}^{\frac{1} {p} } < \infty, \\ 0& < \vert \vert G\vert \vert _{q,\tilde{\psi }} = \left \{\int _{c}^{d}\tilde{\varPsi }(t)G^{q}(t)dt\right \}^{\frac{1} {q} } < \infty, \end{array} & {}\\ \end{array}$$
  1. (i)

    if p > 1, then we have the following equivalent inequalities with the best possible constant factor k λ (σ): 

    $$\displaystyle\begin{array}{rcl} \int _{c}^{d}\int _{ a}^{b}k_{\lambda }(u(s),v(t))f(s)G(t)dsdt < k_{\lambda }(\sigma )\vert \vert f\vert \vert _{ p,\tilde{\phi }}\vert \vert G\vert \vert _{q,\tilde{\psi }},& & {}\end{array}$$
    (66)
    $$\displaystyle\begin{array}{rcl} \left \{\int _{c}^{d}(v(t))^{p\sigma -1}v^{{\prime}}(t)\left (\int _{ a}^{b}k_{\lambda }(u(s),v(t))f(s)ds\right )^{p}dt\right \}^{\frac{1} {p} } < k_{\lambda }(\sigma )\vert \vert f\vert \vert _{p,\tilde{\phi }};& & {}\end{array}$$
    (67)
  2. (ii)

    if 0 < p < 1, or p < 0, there exists a constant δ 0 > 0, such that for any \(\tilde{\sigma }\in (\sigma -\delta _{0},\sigma +\delta _{0})\), \(k_{\lambda }(\tilde{\sigma }) \in \mathbf{R}\), then we still have the equivalent reverses of (66) and (67) with the same best constant factor k λ (σ).

In particular, for \(u_{i}(s_{i}) =\ln s_{i},u_{i}^{{\prime}}(s_{i}) = s_{i}^{-1},u_{i}(1^{+}) = 0,u_{i}(\infty ) = \infty (a_{i} = 1,b_{i} = \infty,i = 1,\ldots,i_{0})\), \(U(s) = (\ln s_{1},\ldots,\ln s_{i_{0}}),v_{j}(t_{j}) =\ln t_{j},v_{j}^{{\prime}}(t_{j}) = t_{j}^{-1},v_{j}(1^{+}) = 0,v_{j}(\infty )\) \(= \infty (c_{j} = 1,d_{j} = \infty,j = 1,\ldots,j_{0})\), \(V (t) = (\ln t_{1},\ldots,\ln t_{j_{0}})\), and

$$\displaystyle\begin{array}{rcl} \tilde{\varPhi }(s)& =& \hat{\varPhi }(s):= \frac{\vert \vert U(s)\vert \vert _{\alpha }^{p(i_{0}-\mu )-i_{0}}} {\left (\varPi _{i=1}^{i_{0}}s_{i}\right )^{1-p}}, {}\\ \tilde{\varPsi }(t)& =& \hat{\varPsi }(t):= \frac{\vert \vert V (t)\vert \vert _{\alpha }^{q(j_{0}-\sigma )-j_{0}}} {\left (\varPi _{j=1}^{j_{0}}t_{j}\right )^{1-q}} {}\\ \end{array}$$

in Corollary 10, we have

Corollary 12.

Suppose that \(\alpha,\beta > 0,\ \lambda,\mu,\sigma \in \mathbf{R},\mu +\sigma =\lambda,k_{\lambda }(x,y)(\geq 0)\) is a homogeneous function of degree −λ in \(\mathbf{R}_{+}^{2}\) , with

$$\displaystyle\begin{array}{rcl} & k_{\lambda }(\sigma ) =\int _{ 0}^{\infty }k_{\lambda }(1,v)v^{\sigma -1}dv \in \mathbf{R}_{+}, & {}\\ & K_{\lambda }(\sigma ) = \left [ \frac{\varGamma ^{j_{0}}\big(\frac{1} {\beta } \big)} {\beta ^{j_{0}-1}\varGamma \big(\frac{j_{0}} {\beta } \big)}\right ]^{\frac{1} {p} }\left [ \frac{\varGamma ^{i_{0}}\big(\frac{1} {\alpha } \big)} {\alpha ^{i_{0}-1}\varGamma \big(\frac{i_{0}} {\alpha } \big)}\right ]^{\frac{1} {q} }k_{\lambda }(\sigma ),& {}\\ \end{array}$$

\(p \in \mathbf{R}\setminus \{0,1\}\),\(\frac{1} {p} + \frac{1} {q} = 1\),\(f(s) = f(s_{1},\ldots,s_{i_{0}}) \geq 0\),\(G(t) = G(t_{1},\ldots,t_{j_{0}}) \geq 0\),

$$\displaystyle\begin{array}{rcl} 0& <& \vert \vert f\vert \vert _{p,\hat{\varPhi }} = \left \{\int _{\left \{s\in \mathbf{R}^{i_{0}};1<s_{i}<\infty \right \}}\hat{\varPhi }(s)f^{p}(s)ds\right \}^{\frac{1} {p} } < \infty, {}\\ 0& <& \vert \vert G\vert \vert _{q,\hat{\varPsi }} = \left \{\int _{\left \{t\in \mathbf{R}^{j_{0}};1<t_{j}<\infty \right \}}\hat{\varPsi }(t)G^{q}(t)dt\right \}^{\frac{1} {q} } < \infty. {}\\ \end{array}$$
  1. (i)

    If p > 1, then we have the following equivalent inequalities with the best possible constant factor K(σ):

    $$\displaystyle\begin{array}{rcl} & \int _{\left \{t\in \mathbf{R}^{j_{0}};1<t_{j}<\infty \right \}}\int _{\left \{s\in \mathbf{R}^{i_{0}};1<s_{i}<\infty \right \}}k_{\lambda }(\vert \vert U(s)\vert \vert _{\alpha },\vert \vert V (t)\vert \vert _{\beta })f(s)G(t)dsdt& \\ & < K_{\lambda }(\sigma )\vert \vert f\vert \vert _{p,\hat{\varPhi }}\vert \vert G\vert \vert _{q,\hat{\varPsi }}, & {}\end{array}$$
    (68)
    $$\displaystyle\begin{array}{rcl} & \left \{\int _{\left \{t\in \mathbf{R}^{j_{0}};1<t_{j}<\infty \right \}}\vert \vert V (t)\vert \vert _{\beta }^{p\sigma -j_{0}}\varPi _{j=1}^{j_{0}}t_{j}^{-1}\left (\int _{\left \{s\in \mathbf{R}^{i_{ 0}};1<s_{i}<\infty \right \}}k_{\lambda }(\vert \vert U(s)\vert \vert _{\alpha },\vert \vert V (t)\vert \vert _{\beta })\right.\right.& \\ & \times \left.\left.f(s)ds\right )^{p}dt\right \}^{\frac{1} {p} } < K_{\lambda }(\sigma )\vert \vert f\vert \vert _{p,\hat{\varPhi }}; & {}\end{array}$$
    (69)
  2. (ii)

    if 0 < p < 1, or p < 0, there exists a constant δ 0 > 0, such that for any \(\tilde{\sigma }\in (\sigma -\delta _{0},\sigma +\delta _{0})\),\(k_{\lambda }(\tilde{\sigma }) \in \mathbf{R}\) , then we still have the equivalent reverses of (68) and (69) with the same best constant factor K λ (σ).

In particular, for i 0 = j 0 = α = β = 1,

$$\displaystyle\begin{array}{rcl} & \tilde{\phi }(s) =\hat{\phi } (s):= \frac{(\ln s)^{p(1-\mu )-1}} {s^{1-p}},\tilde{\psi }(t) =\hat{\psi } (t):= \frac{(\ln t)^{q(1-\sigma )-1}} {t^{1-q}}, & {}\\ & \begin{array}{ll} 0& < \vert \vert f\vert \vert _{p,\hat{\phi }} = \left \{\int _{1}^{\infty }\hat{\phi }(s)f^{p}(s)ds\right \}^{\frac{1} {p} } < \infty, \\ 0& < \vert \vert G\vert \vert _{q,\hat{\psi }} = \left \{\int _{1}^{\infty }\hat{\psi }(t)G^{q}(t)dt\right \}^{\frac{1} {q} } < \infty,\end{array} & {}\\ \end{array}$$
  1. (i)

    if p > 1, then we have the following equivalent inequalities with the best possible constant factor k λ (σ): 

    $$\displaystyle\begin{array}{rcl} \int _{1}^{\infty }\int _{ 1}^{\infty }k_{\lambda }(\ln s,\ln t)f(s)G(t)dsdt < k_{\lambda }(\sigma )\vert \vert f\vert \vert _{ p,\hat{\phi }}\vert \vert G\vert \vert _{q,\hat{\psi }},& & {}\end{array}$$
    (70)
    $$\displaystyle\begin{array}{rcl} \left \{\int _{1}^{\infty }(\ln t)^{p\sigma -1}\frac{1} {t}\left (\int _{1}^{\infty }k_{\lambda }(\ln s,\ln t)f(s)ds\right )^{p}dt\right \}^{\frac{1} {p} } < k_{\lambda }(\sigma )\vert \vert f\vert \vert _{p,\hat{\phi }};& & {}\end{array}$$
    (71)
  2. (ii)

    if 0 < p < 1, or p < 0, there exists a constant δ 0 > 0, such that for any \(\tilde{\sigma }\in (\sigma -\delta _{0},\sigma +\delta _{0})\), \(k_{\lambda }(\tilde{\sigma }) \in \mathbf{R}\), then we still have the equivalent reverses of (70) and (71) with the same best constant factor k λ (σ).

7 Some Particular Examples on the Norm

Example 1.

For \(h(v) = \frac{\vert \ln v\vert ^{\gamma }} {(1+v)^{\lambda }}(\gamma \geq 0,\mu,\sigma > 0,\mu +\sigma =\lambda )\), we have

$$\displaystyle{k(\sigma ) = k_{\gamma }(\sigma ):=\int _{ 0}^{\infty } \frac{\vert \ln v\vert ^{\gamma }} {(1 + v)^{\lambda }}v^{\sigma -1}dv.}$$

Since \(\frac{\vert \ln v\vert ^{\gamma }} {(1+v)^{\lambda /2}} v^{ \frac{\sigma }{ 2} } \rightarrow 0(v \rightarrow 0^{+}\) or v → ), there exists a constant number L > 0, such that

$$\displaystyle{0 < \frac{\vert \ln v\vert ^{\gamma }} {(1 + v)^{\lambda /2}}v^{ \frac{\sigma }{ 2} } \leq L(v \in \mathbf{R}_{+}).}$$

Then it follows that

$$\displaystyle{0 < k_{\gamma }(\sigma ) \leq L\int _{0}^{\infty }\frac{v^{(\sigma /2)-1}dv} {(1 + v)^{\lambda /2}} = LB\left ( \frac{\sigma } {2}, \frac{\mu } {2}\right ) < \infty,}$$

and \(k_{\gamma }(\sigma ) \in \mathbf{R}_{+}\). We find

$$\displaystyle{ k_{0}(\sigma ) =\int _{ 0}^{\infty } \frac{1} {(1 + v)^{\lambda }}v^{\sigma -1}dv = B(\sigma,\mu ). }$$
(72)

For γ ≥ 0, we obtain

$$\displaystyle\begin{array}{rcl} & \begin{array}{ll} k_{\gamma }(\sigma )& =\int _{ 0}^{1}\frac{(-\ln v)^{\gamma }v^{\sigma -1}} {(1+v)^{\lambda }} dv +\int _{ 1}^{\infty }\frac{(\ln v)^{\gamma }v^{\sigma -1}} {(1+v)^{\lambda }} dv \\ & =\int _{ 0}^{1} \frac{(-\ln v)^{\gamma }} {(1+v)^{\lambda }}\left (v^{\sigma -1} + v^{\mu -1}\right )dv \end{array} & {}\\ & \begin{array}{ll} & =\int _{ 0}^{1}(-\ln v)^{\gamma }\sum _{k=0}^{\infty }\binom{-\lambda }{k}\left (v^{k+\sigma -1} + v^{k+\mu -1}\right )dv \\ & =\sum _{ k=0}^{\infty }\binom{-\lambda }{k}\int _{0}^{1}\left (-\ln v\right )^{\gamma }\left (v^{k+\sigma -1} + v^{k+\mu -1}\right )dv.\end{array} & {}\\ \end{array}$$

Setting t = −lnv, we find

$$\displaystyle\begin{array}{rcl} & k_{\gamma }(\sigma ) =\sum _{ k=0}^{\infty }\binom{-\lambda }{k}\int _{0}^{\infty }t^{(\gamma +1)-1}\left [e^{-t(k+\sigma )} + e^{-t(k+\mu )}\right ]dt& \\ & =\varGamma (\gamma +1)\sum _{k=0}^{\infty }\binom{-\lambda }{k}\left [ \frac{1} {(k+\sigma )^{\gamma +1}} + \frac{1} {(k+\mu )^{\gamma +1}} \right ]. &{}\end{array}$$
(73)

In view of Theorem 1 and (39), we have

$$\displaystyle\begin{array}{rcl} & \vert \vert T\vert \vert = K_{\gamma }(\sigma ):= \left [ \frac{\varGamma ^{j_{0}}\big(\frac{1} {\beta } \big)} {\beta ^{j_{0}-1}\varGamma \big(\frac{j_{0}} {\beta } \big)}\right ]^{\frac{1} {p} }& \\ & \qquad \times \left [ \frac{\varGamma ^{i_{0}}\big(\frac{1} {\alpha } \big)} {\alpha ^{i_{0}-1}\varGamma \big(\frac{i_{0}} {\alpha } \big)}\right ]^{ \frac{1} {,q} }k_{\gamma }(\sigma ). &{}\end{array}$$
(74)

Example 2.

For \(h(v) = \frac{\vert \ln v\vert ^{\gamma }} {1+v^{\lambda }}(\gamma \geq 0,\mu,\sigma > 0,\mu +\sigma =\lambda )\), we have

$$\displaystyle{k(\sigma ) = l_{\gamma }(\sigma ):=\int _{ 0}^{\infty } \frac{\vert \ln v\vert ^{\gamma }} {1 + v^{\lambda }}v^{\sigma -1}dv.}$$

Since \(\frac{\vert \ln v\vert ^{\gamma }} {(1+v^{\lambda })^{1/2}} v^{ \frac{\sigma }{ 2} } \rightarrow 0(v \rightarrow 0^{+}\) or v → ), there exists a constant number L > 0, such that

$$\displaystyle{0 < \frac{\vert \ln v\vert ^{\gamma }} {(1 + v^{\lambda })^{1/2}}v^{ \frac{\sigma }{ 2} } \leq L(v \in \mathbf{R}_{+}).}$$

Then it follows that

$$\displaystyle\begin{array}{rcl} 0& <& l_{\gamma }(\sigma ) \leq L\int _{0}^{\infty }\frac{v^{(\sigma /2)-1}dv} {(1 + v^{\lambda })^{1/2}} {}\\ & =& \frac{L} {\lambda } \int _{0}^{\infty }\frac{u^{(\sigma /2\lambda )-1}dv} {(1 + u)^{1/2}} = \frac{L} {\lambda } B\left ( \frac{\sigma } {2\lambda }, \frac{\mu } {2\lambda }\right ) < \infty, {}\\ \end{array}$$

and \(l_{\gamma }(\sigma ) \in \mathbf{R}_{+}\). We find

$$\displaystyle{ l_{0}(\sigma ) =\int _{ 0}^{\infty } \frac{1} {1 + v^{\lambda }}v^{\sigma -1}dv = \frac{\pi } {\lambda \sin \left (\frac{\pi \sigma }{\lambda }\right )}. }$$
(75)

For γ ≥ 0, we obtain

$$\displaystyle\begin{array}{rcl} l_{\gamma }(\sigma )& =& \int _{0}^{1}\frac{(-\ln v)^{\gamma }v^{\sigma -1}} {1 + v^{\lambda }} dv +\int _{ 1}^{\infty }\frac{(\ln v)^{\gamma }v^{\sigma -1}} {1 + v^{\lambda }} dv {}\\ & =& \int _{0}^{1} \frac{(-\ln v)^{\gamma }} {1 + v^{\lambda }}\left (v^{\sigma -1} + v^{\mu -1}\right )dv {}\\ & =& \int _{0}^{1}(-\ln v)^{\gamma }\sum _{ k=0}^{\infty }(-1)^{k}\left (v^{k\lambda +\sigma -1} + v^{k\lambda +\mu -1}\right )dv {}\\ & =& \sum _{k=0}^{\infty }(-1)^{k}\int _{ 0}^{1}(-\ln v)^{\gamma }\left (v^{k\lambda +\sigma -1} + v^{k\lambda +\mu -1}\right )dv. {}\\ \end{array}$$

Setting t = −lnv, we find

$$\displaystyle\begin{array}{rcl} & l_{\gamma }(\sigma ) =\sum _{ k=0}^{\infty }(-1)^{k}\int _{0}^{\infty }t^{(\gamma +1)-1}\left [e^{-t(k\lambda +\sigma )} + e^{-t(k\lambda +\mu )}\right ]dt& \\ & \qquad \qquad =\varGamma (\gamma +1)\sum _{k=0}^{\infty }(-1)^{k}\left [ \frac{1} {(k\lambda +\sigma )^{\gamma +1}} + \frac{1} {(k\lambda +\mu )^{\gamma +1}} \right ]. &{}\end{array}$$
(76)

In view of Theorem 1 and (39), we have

$$\displaystyle\begin{array}{rcl} & \vert \vert T\vert \vert = L_{\gamma }(\sigma ):= \left [ \frac{\varGamma ^{j_{0}}\big(\frac{1} {\beta } \big)} {\beta ^{j_{0}-1}\varGamma \big(\frac{j_{0}} {\beta } \big)}\right ]^{\frac{1} {p} }& \\ & \qquad \times \left [ \frac{\varGamma ^{i_{0}}\big(\frac{1} {\alpha } \big)} {\alpha ^{i_{0}-1}\varGamma \big(\frac{i_{0}} {\alpha } \big)}\right ]_{\gamma }^{ \frac{1} {,q} }l_{\gamma }(\sigma ). &{}\end{array}$$
(77)

Example 3.

For \(h(v) = \frac{\vert \ln v\vert ^{\gamma }} {(\max \{1,v\})^{\lambda }}(\gamma \geq 0,\mu,\sigma > 0,\mu +\sigma =\lambda )\), we have

$$\displaystyle\begin{array}{rcl} k(\sigma )& =& \int _{0}^{\infty } \frac{\vert \ln v\vert ^{\gamma }} {(\max \{1,v\})^{\lambda }}v^{\sigma -1}dv {}\\ & =& \int _{0}^{1}(-\ln v)^{\gamma }v^{\sigma -1}dv +\int _{ 1}^{\infty }\frac{(\ln v)^{\gamma }} {v^{\lambda }} v^{\sigma -1}dv {}\\ & =& \int _{0}^{1}(-\ln v)^{\gamma }\left (v^{\sigma -1} + v^{\mu -1}\right )dv. {}\\ \end{array}$$

Setting t = −lnv, we find

$$\displaystyle\begin{array}{rcl} & \begin{array}{ll} k(\sigma )& =\int _{ 0}^{\infty }t^{\gamma }\left [e^{-(\sigma -1)t} + e^{-(\mu -1)t}\right ]e^{-t}dt \\ & =\int _{ 0}^{\infty }t^{(\gamma +1)-1}\left (e^{-\sigma t} + e^{-\mu t}\right )dt\end{array} & \\ & \qquad \quad =\varGamma (\gamma +1)\left ( \frac{1} {\sigma ^{\gamma +1}} + \frac{1} {\mu ^{\gamma +1}} \right ) \in \mathbf{R}_{+}. &{}\end{array}$$
(78)

In view of Theorem 1 and (39), we have

$$\displaystyle\begin{array}{rcl} & \vert \vert T\vert \vert = K(\sigma ) = \left [ \frac{\varGamma ^{j_{0}}\big(\frac{1} {\beta } \big)} {\beta ^{j_{0}-1}\varGamma \big(\frac{j_{0}} {\beta } \big)}\right ]^{\frac{1} {p} } & \\ & \qquad \qquad \qquad \qquad \qquad \qquad \quad \times \left [ \frac{\varGamma ^{i_{0}}\big(\frac{1} {\alpha } \big)} {\alpha ^{i_{0}-1}\varGamma \big(\frac{i_{0}} {\alpha } \big)}\right ]_{\gamma }^{ \frac{1} {,q} }\varGamma (\gamma +1)\left ( \frac{1} {\sigma ^{\gamma +1}} + \frac{1} {\mu ^{\gamma +1}} \right ).&{}\end{array}$$
(79)

Example 4.

For \(h(v) = \frac{\vert \ln v\vert ^{\gamma }} {\vert 1-v\vert ^{\lambda }}(\gamma \geq 0,\mu,\sigma > 0,\mu +\sigma =\lambda < 1)\), we have

$$\displaystyle{k(\sigma ) =\tilde{ k}_{\gamma }(\sigma ):=\int _{ 0}^{\infty } \frac{\vert \ln v\vert ^{\gamma }} {\vert 1 - v\vert ^{\lambda }}v^{\sigma -1}dv.}$$

We find

$$\displaystyle\begin{array}{rcl} \tilde{k}_{0}(\sigma )& =& \int _{0}^{\infty } \frac{v^{\sigma -1}} {\vert 1 - v\vert ^{\lambda }}dv \\ & =& \int _{0}^{1}(1 - v)^{-\lambda }v^{\sigma -1}dv +\int _{ 1}^{\infty } \frac{v^{\sigma -1}} {(v - 1)^{\lambda }}dv \\ & =& \int _{0}^{1}(1 - v)^{(1-\lambda )-1}v^{\sigma -1}dv +\int _{ 0}^{1}(1 - u)^{(1-\lambda )-1}u^{\mu -1}du \\ & =& B(1-\lambda,\sigma ) + B(1-\lambda,\mu ). {}\end{array}$$
(80)

For γ ≥ 0, we obtain

$$\displaystyle\begin{array}{rcl} \tilde{k}_{\gamma }(\sigma )& =& \int _{0}^{1}\frac{(-\ln v)^{\gamma }v^{\sigma -1}} {(1 - v)^{\lambda }} dv +\int _{ 1}^{\infty }\frac{(\ln v)^{\gamma }v^{\sigma -1}} {(v - 1)^{\lambda }}dv {}\\ & =& \int _{0}^{1} \frac{(-\ln v)^{\gamma }} {(1 - v)^{\lambda }}\left (v^{\sigma -1} + v^{\mu -1}\right )dv. {}\\ \end{array}$$

Setting \(0 <\delta <\min \{\mu,\sigma \}\), since \((-\ln v)^{\gamma }v^{\delta } \rightarrow 0(v \rightarrow 0^{+})\), there exists a constant L > 0, such that \(0 < (-\ln v)^{\gamma }v^{\delta } \leq L(v \in (0,1])\), and then it follows

$$\displaystyle\begin{array}{rcl} 0& <& \tilde{k}_{\gamma }(\sigma ) \leq L\int _{0}^{1}\frac{v^{\sigma -\delta -1} + v^{\mu -\delta -1}} {(1 - v)^{\lambda }} dv {}\\ & =& L(B(1-\lambda,\sigma -\delta ) + B(1-\lambda,\mu -\delta ). {}\\ \end{array}$$

Hence \(\tilde{k}_{\gamma }(\sigma ) \in \mathbf{R}_{+}\), and

$$\displaystyle\begin{array}{rcl} \tilde{k}_{\gamma }(\sigma )& =& \int _{0}^{1}(-\ln v)^{\gamma }\sum _{ k=0}^{\infty }(-1)^{k}\binom{-\lambda }{k}\left (v^{k+\sigma -1} + v^{k+\mu -1}\right )dv {}\\ & =& \sum _{k=0}^{\infty }(-1)^{k}\binom{-\lambda }{k}\int _{ 0}^{1}(-\ln v)^{\gamma }\left (v^{k+\sigma -1} + v^{k+\mu -1}\right )dv. {}\\ \end{array}$$

Setting t = −lnv, we find

$$\displaystyle\begin{array}{rcl} & \tilde{k}_{\gamma }(\sigma ) =\sum _{ k=0}^{\infty }(-1)^{k}\binom{-\lambda }{k}\int _{0}^{\infty }t^{(\gamma +1)-1}\left [e^{-t(k+\sigma )} + e^{-t(k+\mu )}\right ]dt& \\ & \qquad \qquad =\varGamma (\gamma +1)\sum _{k=0}^{\infty }(-1)^{k}\binom{-\lambda }{k}\left [ \frac{1} {(k+\sigma )^{\gamma +1}} + \frac{1} {(k+\mu )^{\gamma +1}} \right ]. &{}\end{array}$$
(81)

In view of Theorem 1 and (39), we have

$$\displaystyle\begin{array}{rcl} & \vert \vert T\vert \vert =\tilde{ K}_{\gamma }(\sigma ):= \left [ \frac{\varGamma ^{j_{0}}\big(\frac{1} {\beta } \big)} {\beta ^{j_{0}-1}\varGamma \big(\frac{j_{0}} {\beta } \big)}\right ]^{\frac{1} {p} }& \\ & \qquad \qquad \qquad \times \left [ \frac{\varGamma ^{i_{0}}\big(\frac{1} {\alpha } \big)} {\alpha ^{i_{0}-1}\varGamma \big(\frac{i_{0}} {\alpha } \big)}\right ]^{ \frac{1} {,q} }\tilde{k}_{\gamma }(\sigma )(\gamma \geq 0). &{}\end{array}$$
(82)

Example 5.

For \(h(v) = \frac{\vert \ln v\vert ^{\gamma }} {\vert v^{\lambda }-1\vert }(\gamma > 0,\mu,\sigma > 0,\mu +\sigma =\lambda )\), we have

$$\displaystyle{k(\sigma ) =\hat{ k}_{\gamma }(\sigma ):=\int _{ 0}^{\infty } \frac{\vert \ln v\vert ^{\gamma }} {\vert v^{\lambda } - 1\vert }v^{\sigma -1}dv.}$$

We find

$$\displaystyle\begin{array}{rcl} & \hat{k}_{1}(\sigma ) =\int _{ 0}^{\infty }\frac{(\ln v)v^{\sigma -1}} {v^{\lambda }-1} dv & \\ & \qquad \qquad \qquad = \frac{1} {\lambda ^{2}} \int _{0}^{\infty }\frac{(\ln u)u^{(\sigma /\lambda )-1}du} {u-1} = [ \frac{\pi } {\lambda \sin \left (\frac{\pi \sigma }{\lambda }\right )}]^{2}.&{}\end{array}$$
(83)

For γ > 0, we obtain

$$\displaystyle\begin{array}{rcl} \hat{k}_{\gamma }(\sigma )& =& \int _{0}^{1}\frac{(-\ln v)^{\gamma }v^{\sigma -1}} {1 - v^{\lambda }} dv +\int _{ 1}^{\infty }\frac{(\ln v)^{\gamma }v^{\sigma -1}} {v^{\lambda } - 1} dv {}\\ & =& \int _{0}^{1} \frac{(-\ln v)^{\gamma }} {1 - v^{\lambda }}\left (v^{\sigma -1} + v^{\mu -1}\right )dv {}\\ \end{array}$$
$$\displaystyle\begin{array}{rcl} \quad \quad =\int _{ 0}^{1}(-\ln v)^{\gamma }\sum _{ k=0}^{\infty }\left (v^{k\lambda +\sigma -1} + v^{k\lambda +\mu -1}\right )dv& & {}\\ \quad \quad =\sum _{ k=0}^{\infty }\int _{ 0}^{1}(-\ln v)^{\gamma }\left (v^{k\lambda +\sigma -1} + v^{k\lambda +\mu -1}\right )dv.& & {}\\ \end{array}$$

Setting t = −lnv, we find

$$\displaystyle\begin{array}{rcl} & \hat{k}_{\gamma }(\sigma ) =\sum _{ k=0}^{\infty }\int _{0}^{\infty }t^{(\gamma +1)-1}\left [e^{-t(k\lambda +\sigma )} + e^{-t(k\lambda +\mu )}\right ]dt& \\ & \qquad \qquad \quad \quad \quad =\varGamma (\gamma +1)\sum _{k=0}^{\infty }\left [ \frac{1} {(k\lambda +\sigma )^{\gamma +1}} + \frac{1} {(k\lambda +\mu )^{\gamma +1}} \right ] \in \mathbf{R}_{+}. &{}\end{array}$$
(84)

In view of Theorem 1 and (39), we have

$$\displaystyle\begin{array}{rcl} & \vert \vert T\vert \vert =\hat{ K}_{\gamma }(\sigma ):= \left [ \frac{\varGamma ^{j_{0}}\big(\frac{1} {\beta } \big)} {\beta ^{j_{0}-1}\varGamma \big(\frac{j_{0}} {\beta } \big)}\right ]^{\frac{1} {p} }& \\ & \quad \quad \times \left [ \frac{\varGamma ^{i_{0}}\big(\frac{1} {\alpha } \big)} {\alpha ^{i_{0}-1}\varGamma \big(\frac{i_{0}} {\alpha } \big)}\right ]^{ \frac{1} {,q} }\hat{k}_{\gamma }(\sigma ). &{}\end{array}$$
(85)

Lemma 6.

If C is the set of complex numbers and \(\mathbf{C}_{\infty } = \mathbf{C} \cup \{\infty \}\),\(z_{k} \in \mathbf{C}\setminus \{z\vert Rez \geq 0\) , Imz = 0}(k = 1,2,…,n) are different points, the function f(z) is analytic in C except for z i (i = 1,2,…,n), and z = ∞ is a zero point of f(z) whose order is not less than 1, then for α ∈ R , we have

$$\displaystyle{ \int _{0}^{\infty }f(x)x^{\alpha -1}dx = \frac{2\pi i} {1 - e^{2\pi \alpha i}}\sum _{k=1}^{n}Res[f(z)z^{\alpha -1},z_{ k}], }$$
(86)

where \(0 < Im\ln z =\arg z < 2\pi\) . In particular, if z k (k = 1,…,n) are all poles of order 1, setting \(\varphi _{k}(z) = (z - z_{k})f(z)(\varphi _{k}(z_{k})\neq 0)\) , then

$$\displaystyle{ \int _{0}^{\infty }f(x)x^{\alpha -1}dx = \frac{\pi } {\sin \pi \alpha }\sum _{k=1}^{n}(-z_{ k})^{\alpha -1}\varphi _{ k}(z_{k}). }$$
(87)

Proof.

By Pan et al. [22, p. 118], we have (86). We find

$$\displaystyle\begin{array}{rcl} 1 - e^{2\pi \alpha i}& =& 1 -\cos 2\pi \alpha - i\sin 2\pi \alpha {}\\ & =& -2i\sin \pi \alpha (\cos \pi \alpha +i\sin \pi \alpha ) {}\\ & =& -2ie^{i\pi \alpha }\sin \pi \alpha. {}\\ \end{array}$$

In particular, since \(f(z)z^{\alpha -1} = \frac{1} {z-z_{k}}(\varphi _{k}(z)z^{\alpha -1})\), it is obvious that

$$\displaystyle{Res[f(z)z^{\alpha -1},-a_{ k}] = z_{k}^{\alpha -1}\varphi _{ k}(z_{k}) = -e^{i\pi \alpha }(-z_{ k})^{\alpha -1}\varphi _{ k}(z_{k}).}$$

Then by (86), we obtain (87). The lemma is proved.

Example 6.

For s ∈ N, 0 < a 1 < ⋯ < a s , we set

$$\displaystyle{h(v) = \frac{1} {\prod _{k=1}^{s}(v^{\lambda /s} + a_{k})}(0 <\sigma <\lambda )}$$

By (87), setting u = v λs, we find

$$\displaystyle\begin{array}{rcl} k(\sigma )& =& k_{s}(\sigma ):=\int _{ 0}^{\infty } \frac{1} {\prod _{k=1}^{s}\left (v^{\lambda /s} + a_{k}\right )}v^{\sigma -1}dv {}\\ & =& \frac{s} {\lambda } \int _{0}^{\infty } \frac{1} {\prod _{k=1}^{s}\left (u + a_{k}\right )}u^{\frac{s\sigma } {\lambda } -1}du {}\\ \end{array}$$
$$\displaystyle{ = \frac{\pi s} {\lambda \sin \left (\frac{\pi s\sigma } {\lambda } \right )}\sum _{k=1}^{s}a_{ k}^{\frac{s\sigma } {\lambda } -1}\prod _{j=1(j\neq k)}^{s} \frac{1} {a_{j} - a_{k}} \in \mathbf{R}_{+}. }$$
(88)

In view of Theorem 1 and (39), we have

$$\displaystyle\begin{array}{rcl} & \vert \vert T\vert \vert = K_{s}(\sigma ):= \left [ \frac{\varGamma ^{j_{0}}\big(\frac{1} {\beta } \big)} {\beta ^{j_{0}-1}\varGamma \big(\frac{j_{0}} {\beta } \big)}\right ]^{\frac{1} {p} }& \\ & \qquad \times \left [ \frac{\varGamma ^{i_{0}}\big(\frac{1} {\alpha } \big)} {\alpha ^{i_{0}-1}\varGamma \big(\frac{i_{0}} {\alpha } \big)}\right ]^{ \frac{1} {,q} }k_{s}(\sigma ). &{}\end{array}$$
(89)

Example 7.

For c > 0, 0 < γ < π, We set

$$\displaystyle{h(v) = \frac{1} {v^{\lambda } + \sqrt{c}v^{\lambda /2}\cos \gamma + \frac{c} {4}}(0 <\sigma <\lambda ).}$$

Putting \(z_{1} = -\frac{\sqrt{c}} {2} e^{i\gamma }\), \(z_{2} = -\frac{\sqrt{c}} {2} e^{-i\gamma }\), by (87), it follows

$$\displaystyle\begin{array}{rcl} k(\sigma )& =& c_{\gamma }(\sigma ):=\int _{ 0}^{\infty } \frac{v^{\sigma -1}} {v^{\lambda } + \sqrt{c}v^{\lambda /2}\cos \gamma + \frac{c} {4}}dv \\ & =& \frac{2} {\lambda } \int _{0}^{\infty } \frac{u^{\frac{2\sigma } {\lambda } -1}} {u^{2} + \sqrt{c}u\cos \gamma + \frac{c} {4}}du \\ & =& \frac{2} {\lambda } \int _{0}^{\infty } \frac{u^{\frac{2\sigma } {\lambda } -1}} {(u - z_{1})(u - z_{2})}du \\ & =& \frac{2\pi } {\lambda \sin \left (\frac{2\pi \sigma } {\lambda } \right )}\left [\left (\frac{\sqrt{c}} {2} e^{i\gamma }\right )^{\frac{2\sigma } {\lambda } -1} \frac{\sqrt{c}} {2(e^{-i\gamma } - e^{i\gamma })}\right. \\ & & \qquad \qquad \qquad \left.+\left (\frac{\sqrt{c}} {2} e^{-i\gamma }\right )^{\frac{2\sigma } {\lambda } -1} \frac{\sqrt{c}} {2(e^{i\gamma } - e^{-i\gamma })}\right ] \\ & =& \left (\frac{\sqrt{c}} {2} \right )^{\frac{2\sigma } {\lambda } } \frac{2\pi \sin \gamma \left (1 -\frac{2\sigma } {\lambda } \right )} {\lambda \sin \gamma \sin \left (\frac{2\pi \sigma } {\lambda } \right )} \in \mathbf{R}_{+}\mathbf{.} {}\end{array}$$
(90)

In view of Theorem 1 and (39), we have

$$\displaystyle\begin{array}{rcl} & \vert \vert T\vert \vert = C_{\gamma }(\sigma ):= \left [ \frac{\varGamma ^{j_{0}}\big(\frac{1} {\beta } \big)} {\beta ^{j_{0}-1}\varGamma \big(\frac{j_{0}} {\beta } \big)}\right ]^{\frac{1} {p} }& \\ & \qquad \times \left [ \frac{\varGamma ^{i_{0}}\big(\frac{1} {\alpha } \big)} {\alpha ^{i_{0}-1}\varGamma \big(\frac{i_{0}} {\alpha } \big)}\right ]^{ \frac{1} {,q} }c_{\gamma }(\sigma ). &{}\end{array}$$
(91)

Example 8.

We set

$$\displaystyle{h(v) = \frac{(\min \{v,1\})^{\eta }} {(\max \{v,1\})^{\lambda +\eta }}(\eta > -\min \{\sigma,\mu \},\sigma +\mu =\lambda ).}$$

Then we find

$$\displaystyle\begin{array}{rcl} k(\sigma )& =& \int _{0}^{\infty }\frac{(\min \{v,1\})^{\eta }v^{\sigma -1}} {(\max \{v,1\})^{\lambda +\eta }} dv =\int _{ 0}^{1}v^{\eta +\sigma -1}dv +\int _{ 1}^{\infty }\frac{v^{\sigma -1}dv} {v^{\lambda +\eta }} \\ & =& \frac{\lambda +2\eta } {(\sigma +\eta )(\mu +\eta )} \in \mathbf{R}_{+}. {}\end{array}$$
(92)

In view of Theorem 1 and (39), we have

$$\displaystyle\begin{array}{rcl} & \vert \vert T\vert \vert = K_{\eta }(\sigma ):= \left [ \frac{\varGamma ^{j_{0}}\big(\frac{1} {\beta } \big)} {\beta ^{j_{0}-1}\varGamma \big(\frac{j_{0}} {\beta } \big)}\right ]^{\frac{1} {p} }& \\ & \qquad \qquad \qquad \times \left [ \frac{\varGamma ^{i_{0}}\big(\frac{1} {\alpha } \big)} {\alpha ^{i_{0}-1}\varGamma \big(\frac{i_{0}} {\alpha } \big)}\right ]^{ \frac{1} {,q} } \frac{\lambda +2\eta } {(\sigma +\eta )(\mu +\eta )}. &{}\end{array}$$
(93)

Example 9.

We set

$$\displaystyle{h(v) =\ln \left ( \frac{b + v^{\gamma }} {a + v^{\gamma }}\right )(0 \leq a < b,0 <\sigma <\gamma ).}$$

We find

$$\displaystyle\begin{array}{rcl} k(\sigma )& =& \int _{0}^{\infty }\ln \left ( \frac{b + v^{\gamma }} {a + v^{\gamma }}\right )v^{\sigma -1}dv {}\\ & =& \frac{1} {\sigma } \int _{0}^{\infty }\ln \left ( \frac{b + v^{\gamma }} {a + v^{\gamma }}\right )dv^{\sigma } {}\\ & =& \frac{1} {\sigma } \left [v^{\sigma }\ln \left ( \frac{b + v^{\gamma }} {a + v^{\gamma }}\right )\vert _{0}^{\infty }\right. {}\\ & & \quad \quad \left.+\gamma \int _{0}^{\infty }\left ( \frac{1} {a + v^{\gamma }} - \frac{1} {b + v^{\gamma }}\right )v^{\sigma +\gamma -1}dv\right ] {}\\ & =& \frac{b - a} {\sigma } \int _{0}^{\infty } \frac{u^{\left (1+\frac{\sigma }{\gamma }\right )-1}} {(u + a)(u + b)}du. {}\\ \end{array}$$

For a > 0, by (87), we have

$$\displaystyle\begin{array}{rcl} k(\sigma )& =& \frac{(b - a)\pi } {\sigma \sin \pi \big(1 + \frac{\sigma }{\gamma }\big)} \left ( \frac{a^{\frac{\sigma }{\gamma }}} {b - a} + \frac{b^{\frac{\sigma }{\gamma }}} {-b + a}\right ) \\ & =& \frac{\left (b^{\frac{\sigma }{\gamma }} - a^{\frac{\sigma }{\gamma }}\right )\pi } {\sigma \sin \left (\frac{\pi \sigma }{\gamma }\right )} \in \mathbf{R}_{+}. {}\end{array}$$
(94)

By using the simple way, we still can obtain (94) for a = 0.

In view of Theorem 1 and (39), we have

$$\displaystyle\begin{array}{rcl} & \vert \vert T\vert \vert = K(\sigma ):= \left [ \frac{\varGamma ^{j_{0}}\big(\frac{1} {\beta } \big)} {\beta ^{j_{0}-1}\varGamma \big(\frac{j_{0}} {\beta } \big)}\right ]^{\frac{1} {p} }& \\ & \qquad \qquad \qquad \times \left [ \frac{\varGamma ^{i_{0}}\big(\frac{1} {\alpha } \big)} {\alpha ^{i_{0}-1}\varGamma \big(\frac{i_{0}} {\alpha } \big)}\right ]^{ \frac{1} {,q} }\frac{\left (b^{\frac{\sigma }{\gamma } }-a^{\frac{\sigma }{\gamma } }\right )\pi } {\sigma \sin \left (\frac{\pi \sigma }{\gamma }\right )}. &{}\end{array}$$
(95)

Example 10.

We set

$$\displaystyle{h(v) = e^{-\rho v^{\gamma }}(\rho,\gamma,\sigma > 0).}$$

Setting u = ρ v γ, we find

$$\displaystyle\begin{array}{rcl} k(\sigma )& =& \int _{0}^{\infty }e^{-\rho v^{\gamma }}v^{\sigma -1}dv = \frac{1} {\gamma e^{\sigma /\gamma }}\int _{0}^{\infty }e^{-u}u^{\frac{\sigma }{\gamma }-1}du \\ & =& \frac{1} {\gamma \rho ^{\sigma /\gamma }}\varGamma \left (\frac{\sigma } {\gamma }\right ) \in \mathbf{R}_{+}\mathbf{.} {}\end{array}$$
(96)

In view of Theorem 1 and (39), we have

$$\displaystyle\begin{array}{rcl} & \vert \vert T\vert \vert = K(\sigma ):= \left [ \frac{\varGamma ^{j_{0}}\big(\frac{1} {\beta } \big)} {\beta ^{j_{0}-1}\varGamma \big(\frac{j_{0}} {\beta } \big)}\right ]^{\frac{1} {p} }& \\ & \qquad \qquad \quad \times \left [ \frac{\varGamma ^{i_{0}}\big(\frac{1} {\alpha } \big)} {\alpha ^{i_{0}-1}\varGamma (\frac{i_{0}} {\alpha } )}\right ]^{ \frac{1} {,q} }\frac{1} {\gamma \rho ^{\sigma /\gamma }}\varGamma \big(\frac{\sigma }{\gamma }\big). &{}\end{array}$$
(97)

Example 11.

We set

$$\displaystyle{h(v) =\arctan \rho v^{-\gamma }(\rho > 0,0 <\sigma <\gamma ).}$$

We find

$$\displaystyle\begin{array}{rcl} k(\sigma )& =& \int _{0}^{\infty }v^{\sigma -1}(\arctan \rho v^{-\gamma })dv {}\\ & =& \frac{1} {\sigma } \int _{0}^{\infty }(\arctan \rho v^{-\gamma })dv^{\sigma } {}\\ & =& \frac{1} {\sigma } \left [(\arctan \rho v^{-\gamma })v^{\sigma }\vert _{ 0}^{\infty } +\int _{ 0}^{\infty } \frac{\gamma \rho v^{\sigma -\gamma -1}} {1 + (\rho v^{-\gamma })^{2}}dv\right ] {}\\ \end{array}$$
$$\displaystyle\begin{array}{rcl} & = \frac{\rho ^{\frac{\sigma }{\gamma } }} {2\sigma }\int _{0}^{\infty } \frac{1} {1+u}u^{\left (\frac{1} {2} -\frac{\sigma }{ 2\gamma } \right )-1}du & \\ & = \frac{\rho ^{\frac{\sigma }{\gamma } }} {2\sigma } \frac{\pi } {\sin \pi \left (\frac{1} {2} -\frac{\sigma }{ 2\gamma } \right )} = \frac{\rho ^{\frac{\sigma }{\gamma } }\pi } {2\sigma \cos \left ( \frac{\pi \sigma }{2\gamma } \right )} \in \mathbf{R}_{+}\mathbf{,}&{}\end{array}$$
(98)

In view of Theorem 1 and (39), we have

$$\displaystyle\begin{array}{rcl} & \vert \vert T\vert \vert = K(\sigma ):= \left [ \frac{\varGamma ^{j_{0}}\big(\frac{1} {\beta } \big)} {\beta ^{j_{0}-1}\varGamma \big(\frac{j_{0}} {\beta } \big)}\right ]^{\frac{1} {p} }& \\ & \qquad \qquad \qquad \times \left [ \frac{\varGamma ^{i_{0}}\big(\frac{1} {\alpha } \big)} {\alpha ^{i_{0}-1}\varGamma \big(\frac{i_{0}} {\alpha } \big)}\right ]^{\frac{1} {q} } \frac{\rho ^{\frac{\sigma } {\gamma } }\pi } {2\sigma \cos \pi \left ( \frac{\sigma }{2\gamma } \right )}. &{}\end{array}$$
(99)

Example 12.

We set

$$\displaystyle{h(v) =\csc h(\rho v^{\gamma }) = \frac{2} {e^{\rho v^{\gamma }} - e^{-\rho v^{\gamma }}}(\rho > 0,\sigma >\gamma > 0),}$$

where \(\csc h(u) = \frac{2} {e^{u}-e^{-u}}\) is hyperbolic cosecant function [23]. We find

$$\displaystyle\begin{array}{rcl} k(\sigma )& =& a_{\gamma }(\sigma ):=\int _{ 0}^{\infty }v^{\sigma -1}\csc h(\rho v^{\gamma })dv {}\\ & =& \int _{0}^{\infty } \frac{2v^{\sigma -1}} {e^{\rho v^{\gamma }} - e^{-\rho v^{\gamma }}}dv {}\\ & =& \int _{0}^{\infty }\frac{2v^{\sigma -1}e^{-\rho v^{\gamma }}dv} {1 - e^{-2\rho v^{\gamma }}} = 2\int _{0}^{\infty }v^{\sigma -1}\sum _{ k=0}^{\infty }e^{-(2k+1)\rho v^{\gamma }}dv {}\\ & =& 2\sum _{k=0}^{\infty }\int _{ 0}^{\infty }v^{\sigma -1}e^{-(2k+1)\rho v^{\gamma }}dv. {}\\ \end{array}$$

Setting u = (2k + 1)ρ v γ, we obtain

$$\displaystyle\begin{array}{rcl} a_{\gamma }(\sigma )& =& \frac{2} {\gamma \rho ^{\sigma /\gamma }}\sum _{k=0}^{\infty } \frac{1} {(2k + 1)^{\sigma /\gamma }}\int _{0}^{\infty }u^{\frac{\sigma }{\gamma }-1}e^{-u}du {}\\ & =& \frac{2} {\gamma \rho ^{\sigma /\gamma }}\varGamma \left (\frac{\sigma } {\gamma }\right )\left [\sum _{k=1}^{\infty } \frac{1} {k^{\sigma /\gamma }} -\sum _{k=1}^{\infty } \frac{1} {(2k)^{\sigma /\gamma }}\right ] {}\\ \end{array}$$
$$\displaystyle{ \quad \quad = \frac{2} {\gamma \rho ^{\sigma /\gamma }}\varGamma \left (\frac{\sigma } {\gamma }\right )\left (1 - \frac{1} {2^{\sigma /\gamma }}\right )\zeta \left (\frac{\sigma } {\gamma }\right ) \in \mathbf{R}_{+}, }$$
(100)

where, \(\zeta \left (\frac{\sigma }{\gamma }\right ) =\sum _{ k=1}^{\infty } \frac{1} {k^{\sigma /\gamma }}\left (\frac{\sigma }{\gamma } > 1\right )\) (ζ(⋅ ) is the Riemann’s zeta function [24]).

In view of Theorem 1 and (39), we have

$$\displaystyle\begin{array}{rcl} & \vert \vert T\vert \vert = A_{\gamma }(\sigma ):= \left [ \frac{\varGamma ^{j_{0}}\big(\frac{1} {\beta } \big)} {\beta ^{j_{0}-1}\varGamma \big(\frac{j_{0}} {\beta } \big)}\right ]^{\frac{1} {p} }& \\ & \quad \quad \times \left [ \frac{\varGamma ^{i_{0}}\big(\frac{1} {\alpha } \big)} {\alpha ^{i_{0}-1}\varGamma \big(\frac{i_{0}} {\alpha } \big)}\right ]^{\frac{1} {q} }a_{\gamma }(\sigma ). &{}\end{array}$$
(101)

Example 13.

We set

$$\displaystyle{h(v) =\sec h(\rho v^{\gamma }) = \frac{2} {e^{\rho v^{\gamma }} + e^{-\rho v^{\gamma }}}(\rho,\sigma,\gamma > 0),}$$

where \(\sec h(u) = \frac{2} {e^{u}+e^{-u}}\) is hyperbolic secant function. We find

$$\displaystyle\begin{array}{rcl} k(\sigma )& =& b_{\gamma }(\sigma ):=\int _{ 0}^{\infty }v^{\sigma -1}\sec h(\rho v^{\gamma })dv {}\\ & =& \int _{0}^{\infty } \frac{2v^{\sigma -1}dv} {e^{\rho v^{\gamma }} + e^{-\rho v^{\gamma }}} =\int _{ 0}^{\infty }\frac{2v^{\sigma -1}e^{-\rho v^{\gamma }}dv} {1 + e^{-2\rho v^{\gamma }}} {}\\ & =& 2\int _{0}^{\infty }v^{\sigma -1}\sum _{ k=0}^{\infty }(-1)^{k}e^{-(2k+1)\rho v^{\gamma }}dv {}\\ & =& 2\sum _{k=0}^{\infty }(-1)^{k}\int _{ 0}^{\infty }v^{\sigma -1}e^{-(2k+1)\rho v^{\gamma }}dv. {}\\ \end{array}$$

Setting u = (2k + 1)ρ v γ, we obtain

$$\displaystyle\begin{array}{rcl} & b_{\gamma }(\sigma ) =\, \frac{2} {\gamma \rho ^{\sigma /\gamma }}\sum _{k=0}^{\infty } \frac{(-1)^{k}} {(2k+1)^{\sigma /\gamma }}\int _{0}^{\infty }u^{\frac{\sigma }{\gamma }-1 }e^{-u}du& \\ & \qquad \!\!\! =\, \frac{1} {\gamma \rho ^{\sigma /\gamma }2^{(\sigma /\gamma )-1}} \varGamma \left (\frac{\sigma }{\gamma }\right )\varsigma \left (\frac{\sigma }{\gamma }\right ) \in \mathbf{R}_{+}, &{}\end{array}$$
(102)

where

$$\displaystyle{\varsigma \left (\frac{\sigma } {\gamma }\right ) =\sum _{ k=0}^{\infty } \frac{(-1)^{k}} {(2k + 1)^{\sigma /\gamma }}\left (\frac{\sigma } {\gamma } > 0\right ).}$$

In view of Theorem 1 and (39), we have

$$\displaystyle\begin{array}{rcl} & \vert \vert T\vert \vert = B_{\gamma }(\sigma ):= \left [ \frac{\varGamma ^{j_{0}}\big(\frac{1} {\beta } \big)} {\beta ^{j_{0}-1}\varGamma \big(\frac{j_{0}} {\beta } \big)}\right ]^{\frac{1} {p} }& \\ & \quad \quad \times \left [ \frac{\varGamma ^{i_{0}}\big(\frac{1} {\alpha } \big)} {\alpha ^{i_{0}-1}\varGamma \big(\frac{i_{0}} {\alpha } \big)}\right ]^{\frac{1} {q} }b_{\gamma }(\sigma ). &{}\end{array}$$
(103)

Example 14.

We set

$$\displaystyle\begin{array}{rcl} h(v)& =& \coth h(\rho v^{\gamma }) - 1 = \frac{e^{\rho v^{\gamma } } + e^{-\rho v^{\gamma } }} {e^{\rho v^{\gamma }} - e^{-\rho v^{\gamma }}} - 1 {}\\ & =& \frac{2e^{-\rho v^{\gamma } }} {e^{\rho v^{\gamma }} - e^{-\rho v^{\gamma }}}(\rho > 0,\sigma >\gamma > 0), {}\\ \end{array}$$

where \(\coth h(u) = \frac{e^{u}+e^{-u}} {e^{u}-e^{-u}}\) is hyperbolic cotangent function. We find

$$\displaystyle\begin{array}{rcl} k(\sigma )& =& c_{\gamma }(\sigma ):=\int _{ 0}^{\infty }v^{\sigma -1}(\coth h(\rho v^{\gamma }) - 1)dv {}\\ & =& \int _{0}^{\infty }\frac{2e^{-\rho v^{\gamma }}v^{\sigma -1}} {e^{\rho v^{\gamma }} - e^{-\rho v^{\gamma }}}dv =\int _{ 0}^{\infty }\frac{2e^{-2\rho v^{\gamma }}v^{\sigma -1}} {1 - e^{-2\rho v^{\gamma }}} dv {}\\ & =& 2\int _{0}^{\infty }v^{\sigma -1}\sum _{ k=0}^{\infty }e^{-2(k+1)\rho v^{\gamma }}dv {}\\ & =& 2\sum _{k=1}^{\infty }\int _{ 0}^{\infty }v^{\sigma -1}e^{-2k\rho v^{\gamma }}dv. {}\\ \end{array}$$

Setting u = 2k ρ v γ, we obtain

$$\displaystyle\begin{array}{rcl} c_{\gamma }(\sigma )& =& \frac{2} {\gamma \rho ^{\sigma /\gamma }}\sum _{k=1}^{\infty } \frac{1} {(2k)^{\sigma /\gamma }}\int _{0}^{\infty }u^{\frac{\sigma }{\gamma }-1}e^{-u}du \\ & =& \frac{1} {\gamma \rho ^{\sigma /\gamma }2^{(\sigma /\gamma )-1}}\varGamma \left (\frac{\sigma } {\gamma }\right )\zeta \left (\frac{\sigma } {\gamma }\right ) \in \mathbf{R}_{+}. {}\end{array}$$
(104)

In view of Theorem 1 and (39), we have

$$\displaystyle\begin{array}{rcl} & \vert \vert T\vert \vert = C_{\gamma }(\sigma ):= \left [ \frac{\varGamma ^{j_{0}}\big(\frac{1} {\beta } \big)} {\beta ^{j_{0}-1}\varGamma \big(\frac{j_{0}} {\beta } \big)}\right ]^{\frac{1} {p} }& \\ & \quad \quad \times \left [ \frac{\varGamma ^{i_{0}}\big(\frac{1} {\alpha } \big)} {\alpha ^{i_{0}-1}\varGamma \big(\frac{i_{0}} {\alpha } \big)}\right ]^{\frac{1} {q} }c_{\gamma }(\sigma ). &{}\end{array}$$
(105)

Example 15.

We set

$$\displaystyle\begin{array}{rcl} h(v)& =& 1 -\tan h(\rho v^{\gamma }) = 1 -\frac{e^{\rho v^{\gamma } } - e^{-\rho v^{\gamma } }} {e^{\rho v^{\gamma }} + e^{-\rho v^{\gamma }}} {}\\ & =& \frac{2e^{-\rho v^{\gamma } }} {e^{\rho v^{\gamma }} + e^{-\rho v^{\gamma }}}(\rho,\sigma,\gamma > 0), {}\\ \end{array}$$

where \(\tan h(u) = \frac{e^{u}+e^{-u}} {e^{u}-e^{-u}}\) is hyperbolic tangent function. We find

$$\displaystyle\begin{array}{rcl} k(\sigma )& =& d_{\gamma }(\sigma ):=\int _{ 0}^{\infty }v^{\sigma -1}(1 -\tan h(\rho v^{\gamma }))dv {}\\ & =& \int _{0}^{\infty }\frac{2e^{-\rho v^{\gamma }}v^{\sigma -1}} {e^{\rho v^{\gamma }} + e^{-\rho v^{\gamma }}}dv =\int _{ 0}^{\infty }\frac{2e^{-2\rho v^{\gamma }}v^{\sigma -1}} {1 + e^{-2\rho v^{\gamma }}} dv {}\\ & =& 2\int _{0}^{\infty }v^{\sigma -1}\sum _{ k=0}^{\infty }(-1)^{k}e^{-2(k+1)\rho v^{\gamma }}dv {}\\ & =& 2\sum _{k=1}^{\infty }(-1)^{k-1}\int _{ 0}^{\infty }v^{\sigma -1}e^{-2k\rho v^{\gamma }}dv. {}\\ \end{array}$$

Setting u = 2k ρ v γ, we obtain

$$\displaystyle\begin{array}{rcl} d_{\gamma }(\sigma )& =& \frac{2} {\gamma \rho ^{\sigma /\gamma }}\sum _{k=1}^{\infty }\frac{(-1)^{k-1}} {(2k)^{\sigma /\gamma }} \int _{0}^{\infty }u^{\frac{\sigma }{\gamma }-1}e^{-u}du \\ & =& \frac{1} {\gamma \rho ^{\sigma /\gamma }2^{(\sigma /\gamma )-1}}\varGamma \left (\frac{\sigma } {\gamma }\right )\xi \left (\frac{\sigma } {\gamma }\right ) \in \mathbf{R}_{+}, {}\end{array}$$
(106)

where, \(\xi (\frac{\sigma }{\gamma }):=\sum _{ k=1}^{\infty }\frac{(-1)^{k-1}} {k^{\sigma /\gamma }}\).

In view of Theorem 1 and (39), we have

$$\displaystyle\begin{array}{rcl} & \vert \vert T\vert \vert = D_{\gamma }(\sigma ):= \left [ \frac{\varGamma ^{j_{0}}\big(\frac{1} {\beta } \big)} {\beta ^{j_{0}-1}\varGamma \big(\frac{j_{0}} {\beta } \big)}\right ]^{\frac{1} {p} }& \\ & \quad \quad \times \left [ \frac{\varGamma ^{i_{0}}\big(\frac{1} {\alpha } \big)} {\alpha ^{i_{0}-1}\varGamma \big(\frac{i_{0}} {\alpha } \big)}\right ]^{\frac{1} {q} }d_{\gamma }(\sigma ). &{}\end{array}$$
(107)

Note.

The following references [2431] provide an extensive theory and applications of Analytic Number Theory relating to Riemann’s zeta function that will provide a source study for further research on Hilbert-type inequalities.